Простые задачи по математике: Простейшие задачи по математике

Содержание

Задачи по математике в два действия

 


вернуться к оглавлению задач по темам»


Ничего сложного в математических задачах на два действия нет. При условии, конечно, что ваш ребенок щелкает, как орешки, задачки в одно действие.

 

 

Задачи в два и более действий называют составными.  То есть они состоят из более простых, эдакие задачи внутри задач. Посмотреть приемы решения составных задач можно ТУТ»

 

А сами задачи для тренировки смотрим ниже:

 

1. В трёх тетрадях 60 листов. В первой и второй тетрадях – по 24 листа. Сколько листов в третьей тетради?

 

2. Гусь весит 9 кг, а курица – на 7 кг меньше. Сколько весят гусь и курица вместе?

 

3. На школьной выставке 80 рисунков. 23 из них выполнены фломастерами, 40 карандашами, а остальные – красками. Сколько рисунков, выполненные красками, на школьной выставке?

 

4. В школьный буфет привезли два лотка с булочками. На одном лотке было 40 булочек, на другом – 35. За первую перемену продали 57 булочек. Сколько булочек осталось?

 

5. Вера собирала букет из осенних листьев. Дубовых листочков у нее было 12, осиновых – на 4 меньше, а кленовых столько, сколько дубовых и осиновых вместе. Сколько кленовых листочков в Верином букете?

 

6. К началу учебного года мама купила Наташе 19 новых книжек. Из них 7 было без картинок, а из тех, которые с картинками, половина – учебники.  Сколько учебников мама купила Наташе?

 

7. В субботу в музее побывало 26 учеников из 2 “А” класса, а в воскресенье – на 8 человек больше из 2 “Б” класса. Сколько всего учеников вторых классов побывало в музее за субботу и воскресенье?

 

8. В ларьке было 60 пирожков. До обеда продали 26 пирожков, а после обеда – 32 пирожка. Сколько пирожков не продали?

 

9. Оля решила нарисовать 72 букета. В понедельник она нарисовала 18 букетов, во вторник – 22 букета. Сколько букетов Оля не стала рисовать?

 

10. Около школы посадили 15 кустов сирени,  боярышника – на 5 кустов больше, чем сирени, а черемухи – столько, сколько сирени и боярышника вместе. Сколько кустов черёмухи посадили около школы?

 

11. В парке росло 75 дубов. После урагана оказалось, что 7 дубов погибли. Тогда посадили еще 12 дубов. Сколько дубов стало в парке?

 

12. В танцевальную студию ходят 23 ученика из второго класса, а из третьего – на 5 детей больше. Сколько всего учеников из второго и третьего класса ходят в танцевальную студию?

 

13. Из бидона зачерпнули утром 6 кружек кваса, в обед – еще 5 кружек. После этого в бидоне осталось 14 кружек кваса. Сколько кружек кваса было в бидоне с утра?

 

14. В первой четверти в начальной школе было 65 хорошистов, во второй – на 27 больше, чем в первой. А в третьей четверти – на 22 хорошиста меньше, чем во второй.  Сколько учеников закончили школу без троек в третьей четверти?

 

15. В цехе работает 90 человек. Из них 65 мужчин, а остальные – женщины. На сколько больше в цехе работает мужчин, чем женщин?

 

 

 

 

Простые задачи. Методика их решения

В статье раскрывается сущность простых текстовых задач, методика их решения в начальной школе, а так же сам процесс работы с младшими школьниками по решению текстовых задач на уроках математики. Проводится анализ работы над простой задачей на примере решения задачи на кратное сравнение.

Математика встречается на протяжении всей жизни человека. В связи с этим система образования усовершенствует математическую подготовку школьников с каждым годом, учитывая темп роста научно технического прогресса. Начиная с первых дней пребывания в образовательном учреждении, педагоги знакомят детей с самыми первыми знания в математики и насколько доступно учитель сможет преподнести учебный материал, настолько в дальнейшем учащимся будет легко даваться процесс обучения[1].

С первых дней пребывания в образовательном учреждении учащихся начинают знакомить с задачами. Математические задачи помогают обучающимся применять полученные теоретические знания на практике, а так же выяснить различные связи в окружающем мире, а так же развитию логического мышления.

Простые задачи это задачи на одно действие, вычитание, сложение, умножение или деление. Первоклассники обычно решают задачи на сложение и вычитание простых чисел, во втором классе уже внедряются задачи на умножение и деление.

В современной литературе предлагается множество различных приемов, которые помогут облегчить поиск необходимого решения. Но в тоже время теоретические положения нахождения пути решения задачи остаются мало изученными. Решение текстовых задач традиционно представляет собой трудность для учащихся, причем это касается не только начальной, но и средней и старшей школы[2].

В математики решение задач является одним из основных действий, а учащиеся, которые любят математику, могут и решать различные задачи. Следовательно, педагогу необходимо научить обучающихся решать задачи различной сложности, так как это поможет повлиять на интересы ребенка, мышление и речь[3].

Простые задачи помогают учащимся разобраться в математических понятиях, и познакомится с различными арифметическими действиями. После того, как школьники научатся решать простые задачи, они с легкостью смогут решать составные, которые в свою очередь состоять из нескольких простых.

Основным вопросом в методике обучения решения считается нахождения правильного способа решения простой задачи. В современной литературе предлагается множество различных приемов, которые помогут облегчить поиск необходимого решения. Но в тоже время теоретические положения нахождения пути решения задачи остаются мало изученными. Решение текстовых задач традиционно представляет собой трудность для учащихся, причем это касается не только начальной, но и средней и старшей школы.

Решение текстовых задач — это сложная деятельность, содержание которой зависит как от конкретной задачи, так и от умений самого ученика.

Рассмотрим более подробно каждый из этапов решения простой задачи. Для примера рассмотрим решение задачи на краткое сравнение:

В парке выкорчевали 6 старых орешников, а вместо них посадили 18 молодых орешников. Во сколько раз больше посадили орешников, чем выкорчевали?

1. Ознакомление с содержанием задачи.

Под ознакомлением с условием задачи подразумевается внимательное чтение и представление условий в реальной жизни. Педагогам особенно важно научить правильно, читать условия задачи, делая акцент на числовых данных, которые определяют действие, а именно: «было», «уехали» и т.д. Вопрос задачи выделяют интонацией.

Для учащегося, который только начинает решать задачи условием задачи могут стать различные предметы, конкретные лица. Задача учителя разъяснить ребёнку, что необходимо представить условие задачи.

2. Поиск решения задачи;

Учащиеся должны выделить величины, которые входят в задачу, данные и то число, которое необходимо найти. Для того чтобы было легче понять условие задачи рекомендуется схематично изображать условия задачи в тетради и на доске. Схема должна отражать все имеющиеся условия, и если она сделана правильно, то текстовая часть задачи не потребуется.

Выкорчевали — 6 шт.

Посадили — 18 шт., на ? больше

После того, как составлена схема и разобрали условие, школьникам предлагают ответить на несколько вопросов: можно ли сразу ответить на вопрос задачи, если можно, то, какое математическое действие необходимо использовать.

3. Решение задачи. При решении простой задачи необходимым условием является составление краткой записи, который позволяет не только работать над задачей, но и отрабатывать навыки её решения. Для начала рекомендуется научить ребенка составлять краткое условие задачи, а затем предложить краткую запись с другими числами и составить задачу. Выделение «условия» и «вопроса» — это базовые знания, которыми должен владеть каждый.

Чтобы узнать во сколько раз больше орешников посадили, нужно общее количество посаженных деревьев поделить на количество выкорчеванных. Таким образом, в 3 раза больше посадили орешников, чем выкорчевали.

Решение:

1) 18 : 6 = 3

Ответ: в 3 раза больше орешников посадили.

4. Проверка решения задачи

Последним этапом над работой с задачей является самопроверка. Самопроверка позволяет проверить решение задачи на наличие ошибок в ответе. Для начала составим обратную задачу на нахождения другого неизвестного числа. Если при решении обратной задачи получ Например:

В парке выкорчевали 6 старых орешников, а вместо них посадили молодые орешники, количество которых составило в 3 раза больше, чем выкорчеванных старых орешников. Сколько посадили молодых орешников?

Далее составляем краткую запись задачи и ее решение.

Выкорчевали — 6 шт.

Посадили — ? шт., на 3 больше

Решение:

1) 6 × 3 = 18 шт.

Ответ: 18 молодых орешников посадили.

Особым моментом в решении задач является проговаривание учащимися условия задачи и способа их решения. Учащийся должен рассказать, как он решал эту задачу и почему он выбрал именно это решение.

Таким образом, одной из важнейших задач в обучении решении задач, является научить обучающихся внимательно читать условие задачи и составлять краткую запись. Для того чтобы закрепить изученный материал, обучающимся предлагают решать подобные задачи.

Математика: уроки, тесты, задания.

Математика: уроки, тесты, задания.





    1. Сравнение предметов





    2. Точка, прямая линия, кривая и отрезок





    3. Особенности многоугольников





    4. Пространственные и временные представления





    5. Объединение предметов в группы и пары





    6. Сравнение (больше, меньше, столько же)





    7. Знаки сравнения и знаки действий






    1. Нумерация.

      Сколько? От 1 до 5





    2. Примеры на сложение и вычитание от 1 до 5





    3. Сравнение чисел от 1 до 5





    4. Текстовые задачи (от 1 до 5)





    5. Задачи на смекалку (от 1 до 5)






    1. Примеры на сумму





    2. Текстовые задачи (сумма)




  1. Переместительный закон сложения





    1. Примеры на разность





    2. Текстовые задачи (разность)




  2. Таблица сложения.

    Числа от 1 до 9





    1. Нумерация. Сколько? От 0 до 10





    2. Примеры от 0 до 10





    3. Сравнение чисел от 0 до 10 и выражений





    4. Текстовые задачи (от 0 до 10)





    5. Задачи на смекалку (от 0 до 10)




  3. Увеличить/уменьшить на…





    1. Мера длины — сантиметр





    2. Мера длины — дециметр




  4. На сколько больше? На сколько меньше?





    1. Счёт десятками





    2. Счёт круглых чисел






    1. Нумерация.

      Сколько? От 11 до 20





    2. Примеры от 11 до 20





    3. Сравнения чисел от 11 до 20





    4. Текстовые задачи (от 11 до 20)





    5. Задачи на смекалку (от 11 до 20)




  1. Числа от 20 до 100.

    Нумерация. Числа и цифры





    1. Сочетательный закон сложения. Скобки





    2. Таблица сложения. Числа от 0 до 18





    3. Вычитаем сумму из числа





    4. Правила сложения и вычитания чисел в пределах 20 с переходом через десяток





    5. Сложение и вычитание чисел в пределах 100 без перехода через десяток





    6. Правила сложения и вычитания чисел в пределах 100 с переходом через десяток





    7. Правила сложения и вычитания чисел в пределах 100






    1. Находим периметр





    2. Решение задач в два действия






    1. Мера длины — метр





    2. Килограмм





    3. Литр






    1. Уравнение (сумма)





    2. Уравнение (разность)






    1. Понятие умножения





    2. Переместительный закон умножения





    3. Умножение на 2 (таблица)





    4. Умножение на 3 (таблица)





    5. Умножение на 4 (таблица)





    6. Умножение на 5 (таблица)




  2. Деление



  3. Чётные и нечётные числа





    1. Выражения без скобок





    2. Выражения со скобками






    1. Узнаём о луче





    2. Фигура угол и его характеристики





    3. Характеристики прямого, тупого и острого углов






    1. Увеличить на.

      .. Увеличить в… Уменьшить на… Уменьшить в…





    2. Больше на… Больше в… Меньше на… Меньше в…






    1. Умножение на 6 (таблица)





    2. Умножение на 7 (таблица)





    3. Умножение на 8 (таблица)





    4. Умножение на 9 (таблица)






    1. Нахождение неизвестного множителя





    2. Нахождение неизвестного делимого





    3. Нахождение неизвестного делителя






    1. Свойства ломаной линии





    2. Треугольники.

      Виды треугольников






    1. Умножение и деление на 0, 1, 10. Деление числа на само себя





    2. Выполняем умножение и деление круглого числа на однозначное число





    3. Правила деления круглого числа на круглое число






    1. Умножаем сумму на число





    2. Умножаем двузначное число на однозначное число






    1. Правила деления суммы на число





    2. Правила деления двузначного числа на однозначное





    3. Правила деления двузначного числа на двузначное





    4. Правила деления с остатком






    1. Находим долю от числа





    2. Сравниваем доли





    3. Находим число по доле






    1. Трёхзначные числа.

      Нумерация





    2. Сложение и вычитание трёхзначных чисел





    3. Выполняем умножение и деление трёхзначного числа на однозначное число





    4. Связь между величинами




  1. Календарь





    1. Нумерация





    2. Правила сложения и вычитания многозначных чисел





    3. Правила сочетательного закона умножения





    4. Умножаем и делим числа на 10, 100, 1000





    5. Круглые числа (умножение и деление)






    1. Единицы измерения времени (час, минута, сутки)





    2. Миллиметр





    3. Километр






    1. Нахождение площади фигуры, прямоугольника





    2. Единицы измерения площади






    1. Умножение на однозначное число.

      Распределительный закон умножения относительно сложения





    2. Умножаем круглое число на однозначное число





    3. Выполняем умножение на круглое число





    4. Выполняем умножение круглых чисел





    5. Выполняем умножение на двузначное число





    6. Выполняем умножение на трёхзначное число






    1. Деление многозначного числа на однозначное число





    2. Деление круглого многозначного числа на однозначное





    3. Деление многозначного числа на 10, 100, 1000 с остатком





    4. Деление многозначного числа с остатком на однозначное число





    5. Выполняем деление трёхзначного числа на двузначное число





    6. Деление с остатком трёхзначного числа на двузначное число





    7. Деление многозначного числа на двузначное число





    8. Деление с остатком на двузначное число





    9. Выполняем деление на трёхзначное число





    10. Деление с остатком на трёхзначное число





    11. Деление круглого многозначного числа на круглое число






    1. Единицы времени.

      Минута. Секунда





    2. Единицы массы и площади. Гектар. Центнер. Тонна






    1. Понятие дроби





    2. Сравниваем дроби





    3. Дроби.

      Нахождение части числа





    4. Дроби. Нахождение числа по его части






    1. Решение задач на нахождение скорости, времени, расстояния





    2. Решение задач на нахождение работы, времени, производительности





    3. Решение задач на нахождение цены, количества, стоимости






    1. Десятичная система счисления.

      Римская нумерация





    2. Числовые и буквенные выражения





    3. Начальные геометрические понятия: прямая, отрезок, луч, ломаная, прямоугольник





    4. Определение координатного луча





    5. Округление чисел.

      Прикидка и оценка результатов вычислений





    6. Законы арифметических действий. Вычисления с многозначными числами





    7. Решение текстовых задач арифметическим способом





    8. Формулы. Уравнения. Упрощение выражений





    9. Математический язык и математическая модель






    1. Деление с остатком.

      Понятие обыкновенной дроби





    2. Основное свойство дроби. Сокращение и расширение дробей





    3. Правильные и неправильные дроби. Смешанные числа. Понятие, запись и чтение





    4. Сравнение обыкновенных дробей





    5. Сложение и вычитание обыкновенных дробей и смешанных чисел





    6. Умножение и деление обыкновенной дроби на натуральное число





    7. Нахождение части от целого и числа по его части





    8. Геометрические понятия: окружность и круг






    1. Угол.

      Измерение углов





    2. Биссектриса угла. Свойство биссектрисы угла





    3. Треугольник. Площадь треугольника





    4. Свойство углов треугольника. Размеры объектов окружающего мира (масштаб)





    5. Расстояния между двумя точками.

      Масштаб. Виды масштаба





    6. Перпендикулярность прямых. Расстояние от точки до прямой. Серединный перпендикуляр






    1. Понятие десятичной дроби. Представление десятичной дроби в виде обыкновенной дроби и наоборот





    2. Десятичные дроби.

      Сравнение





    3. Десятичные дроби. Сложение и вычитание





    4. Десятичные дроби. Умножение





    5. Степень с натуральным показателем





    6. Десятичные дроби.

      Среднее арифметическое, деление на натуральное число





    7. Десятичные дроби. Деление на десятичную дробь





    8. Проценты. Задачи на проценты: нахождение процента от величины и величины по её проценту






    1. Прямоугольный параллелепипед.

      Определение, свойства





    2. Прямоугольный параллелепипед. Развёртка





    3. Прямоугольный параллелепипед. Объём






    1. Делимость натуральных чисел





    2. Признаки делимости на 2, 3, 5, 9, 10





    3. Простые и составные числа.

      Разложение натурального числа на простые множители





    4. Наибольший общий делитель и наименьшее общее кратное






    1. Положительные и отрицательные числа. Определение координатной прямой





    2. Противоположные числа.

      Модуль числа. Целые и рациональные числа





    3. Сравнение рациональных чисел





    4. Сложение рациональных чисел с помощью координатной прямой





    5. Алгебраическая сумма. Свойства





    6. Алгебраическая сумма рациональных чисел с одинаковыми знаками





    7. Алгебраическая сумма рациональных чисел с разными знаками





    8. Умножение и деление рациональных чисел





    9. Умножение и деление обыкновенных дробей





    10. Дробные выражения





    11. Координаты.

      Координатная плоскость, координаты точки






    1. Отношение двух чисел





    2. Пропорция. Основное свойство пропорции





    3. Прямая и обратная пропорциональность





    4. Решение задач с помощью пропорций





    5. Разные задачи






    1. Упрощение выражений, раскрытие скобок





    2. Решение линейных уравнений





    3. Этапы решения линейных уравнений






    1. Начальные понятия и факты курса геометрии





    2. Параллельность прямых





    3. Центральная и осевая симметрия





    4. Окружность и круг.

      Число Пи. Длина окружности. Площадь круга





    5. Наглядные представления о шаре, сфере. Формулы площади поверхности сферы и объёма шара




  1. Коллекция интерактивных моделей




Задачи для 1 — 2 класса

Примеры решения задач по математике

Задача №1

В шкафу стояло 39 книг. Когда на полку переставили несколько книжек, их там осталось 25.

Сколько книг переставили на полку?

Решение:

39 — 25 = 14 (кн.)

Ответ: на полку переставили 14 книг.

Задача №2

Когда со стоянки уехало 4 машины, там осталось ещё 26.

Сколько машин было на стоянке?

Решение:

26 + 4 = 30 (маш.)

Ответ: на стоянке было 30 машин.

Задача №3

У Олега было 15 почтовых марок. 5 марок он использовал.

Сколько марок осталось у Олега?

Решение:

15 — 5 = 10 (мар.)

Ответ: у Олега осталось 10 марок.

Задача №4

На элеватор привезли 45 т ржи, а пшеницы 15 т.

Сколько всего зерна привезли на элеватор?

Решение:

45 + 15 = 60 (т)

Ответ: на элеватор привезли 60 тонн зерна.

Задача №5

Петя собрал по теме «Космос» 25 марок, а по теме «Флот» только 9.

Сколько всего марок в Петиной коллекции?

Решение:

25 + 9 = 31 (мар.)

Ответ: в Петиной коллекции 31 марка.

Задача №6

В ларёк привезли 15 ящиков яблок и 8 ящиков слив.

Сколько всего привезли ящиков с фруктами?

Решение:

15 + 8 = 23 (ящ.)

Ответ: в ларёк привезли 23 ящика с фруктами.

Задача №7

Бабушка посадила 16 кустиков клубники, а внучка 5.

Сколько всего кустиков клубники они посадили?

Решение:

16 + 5 = 21 (кус.)

Ответ: они посадили 21 куст клубники.

Задача №8

В школьную столовую привезли 15 кг картофеля, через 2 дня ещё 10 кг.

Сколько всего килограммов картофеля привезли?

Решение:

15 + 10 = 25 (кг)

Ответ: всего привезли 25 кг картофеля.

 

Задача №9

На стройку привезли 5 ящиков больших гвоздей и 8 ящиков маленьких.

Сколько всего привезли ящиков с гвоздями?

Решение:

5 + 8 = 13 (ящ.)

Ответ: на стройку привезли 13 ящиков с гвоздями.

Задача №10

Дедушка налил в бочку 18 вёдер воды и привёз в сад. Из бочки, под кусты, вылили 7 вёдер воды.

Сколько вёдер воды осталось бочке?

Решение:

18 — 7 = 11 (вёд.)

Ответ: в бочке осталось 11 вёдер воды.

 

Домашнее задание

Чтобы легко и быстро решать аналогичные задачи нужно закрепить знания выполнив несколько заданий самостоятельно. Реши задачу и дай ответ.

Пример 1

На праздник 9 Мая дети читали стихи. Девочки прочитали 7 стихов, а мальчики 9. Сколько всего прочитали стихотворений?

 

Пример 2

Оля помогала маме мыть посуду. Мама помыла 6 тарелок, а Оля помыла 3 чашки. Сколько всего вымыто посуды?

 

Пример 3

К новогоднему празднику ученики украсили школьный класс. Девочки вырезали из бумаги 15 снежинок, а мальчики 20 хлопушек. Сколько всего украшений сделали ученики?

 

Пример 4

В классе на подоконниках окнон стоят 36 горшков с цветками. На первом подоконнике стоят 12 горшков, на втором 14. Сколько горшков на третьем подоконнике?

 

Пример 5

У Коли в аквариуме 18 рыбок – 9 скалярий и 11 барбусов. Коля подарил Ване 4 скалярия, а Пете 4 барбуса. Сколько рыбок каждой породы осталось у Коли?

Если заметили ошибку, выделите фрагмент текста и нажмите Ctrl+Enter

Математические задачи для 2 класса: простые и сложные задания

Математика от Матшарика: математические задачи, примеры и логика! На этой странице есть простые и сложные задачи по математике на логику. Решайте и развивайтесь!

Виды наших задач по математике для 2 класса:

Логическая задачи и задания в которых между цифрами надо ставить знаки (игра).

Математические задачи, длинные и короткие примеры.

Задачи «примеры в алфавитном порядке».

Цифровые загадки и магические квадраты (игра).

Математические задания на логику для 2 класса

Расставь знаки +-:×.

Неразделённые числа принимаются как одно число!

Например:

2 + 1 + 4 + 3 * 6 = 25.


Расставь знаки +, -, : и × чтобы получилось 71.

7  4  9  3  5  8 = 71.

Расставь между цифрами знаки :, +, -, × чтобы получилось 19.

1   7   9   5   2   3 = 19.

Задачи по математике 2 класс

Леля купила в магазине 4 конфеты, 1 упаковку хлопьев, 10 яиц, 2 пакета молока и 5 яблок. Одно яблоко стоит 30 копеек, то, что в 2 раза дешевле 5 яиц. 32 конфеты стоят 48 копеек, то, что на 63 копейки дешевле 1 пачки хлопьев. 1 пакет молока стоит 15 копеек.

Сколько заплатила Леля?

Примечание: 1 рубль – 100 копеек.

87 + 24, 76 – 14 и 14 + 58.

И какой результат?


Реши супер – пример:

15 + 32 – 28 × 2 – 5 + 14 = ?


Расставь знаки :, +, – и × чтобы получилось 204.

17  3  27  15  8  4 = 204.


Реши примеры, и по алфавитному порядку составь слово.

Примечание: примеры выстроены в правильном порядке.

13 + 6, 48 – 27, 7 × 2, 24 : 2 и 3 – 2.


Реши примеры и найди сумму их ответов.

56 – 31, 52 × 4 и 12 + 69.


Реши супер – пример:

19 + (26 – 17) × 3 – 65 = ?


Реши примеры, и по алфавитному порядку узнай слово.

Примечание: примеры выстроены по правильному порядку.

8 : 2, 7 + 14, 5 – 4, 13 × 2 и 60 : 2.


Реши примеры, и по алфавитному порядку узнай слово.

Примечание: примеры выстроены в правильном порядке.

7 + 6, 2 × 5, 57 : 3 и 5 × 4.


Узнай, что надо поставить.

58732 = 7

17584 = 7

85641 = 12

54683 = …?


Реши примеры, и найди сумму их ответов.

86 + 23, 17 × 2, 85 – 37.


Узнай, что надо поставить.

94538 = 15

72943 = 13

82347 = 12

74328 = …?


Магические квадраты

Перед решением зайди в теорию: https://matsharik.ru/info/teorija-i-sovety

Игра выше содержит 3 уровня, в которых нужно решить или построить магический квадрат.

Как научить детей решать задачи по математике: советы именитых педагогов и простых мам

Научить детей решать задачи по математике — дело учителя, но и родители не должны оставаться в стороне, если их чадо «тормозит» в этом вопросе. Одним учебником математики сыт не будешь. Ведь если научить ребенка самостоятельно решать задачи в 1-3 классах, дальше он будет щелкать как семечки не только задачи по математике, но и по физике, химии, геометрии и др. И самое главное — этот навык пригодится ребенку в жизни!

vogazeta.ru

В статье Как научить ребенка математике мы подробно писали, из каких 4 частей состоит любая задача и что нужно сделать в первую очередь, чтобы ребенок понял, чего от него хотят и как ответить на вопрос задачи. Уяснив алгоритм решения задач, ребенок сможет самостоятельно решить практически любую задачу, даже несмотря на то, что они все кажутся такими разными. 

Основные типы задач по математике: краткий конспект

Небольшой ликбез, т.к. далеко не все родители учились в педагогических ВУЗах и владеют методикой преподавания. Пробежимся по теории, чтобы понимать, кто, кому и чего «должен». Зная ключевые моменты, вам будет проще помочь ребенку в решении задач, которые вызывают у него сложности, вы сможете определить, где пробелы в знаниях и что нужно «подтянуть» в каждом конкретном случае.

iqsha.ru

Рассмотрим самые распространенные виды задач в начальных классах.

1. Простые задачи на сложение и вычитание

К этой группе относятся несколько задач, но для всех есть общие рекомендации:

  • Решаются в одно действие.
  • Иногда удобно составить уравнение.
  • На их примере ребенок должен научится выполнять краткую запись. 
  • Если краткого условия недостаточно, нарисовать рисунок. Если не помог рисунок, показываем на конкретных предметах и производим действия с ними.
  • Четко усвоить, что «+» — это прибавить, увеличить, а «-» — уменьшить, отнять, вычесть.
  • Хорошо запомнить компоненты арифметических действий:

слагаемое + слагаемое = сумма
уменьшаемое — вычитаемое = разность

  • Понять разницу между словами «стало» и «осталось». Четко понимать, что значит «на … меньше», «на … больше».
  • Важно понять и запомнить: чтобы узнать, НА СКОЛЬКО одно число больше или меньше другого, нужно из большего числа вычесть меньшее.
  • Важно понять и запомнить: чтобы найти неизвестное слагаемое, нужно из суммы вычесть известное слагаемое.

 

  • Важно понять и запомнить: чтобы найти неизвестное уменьшаемое, нужно к вычитаемому прибавить разность.

 

  • Важно понять и запомнить: чтобы найти неизвестное вычитаемое, нужно из уменьшаемого вычесть разность.

Задачи с косвенным вопросом

Это самые коварные задачи из этой группы. Внимательно прочитайте условие — и поймете почему.

На стоянке у первого подъезда 7 машин. Это на 2 машины больше, чем на стоянке у второго подъезда. сколько машин на стоянке у второго подъезда.

2. Составные задачи на сложение и вычитание

Эти задачи решаются двумя и более действиями.

Есть несколько способов решения:

  • по действиям с пояснениями;
  • по действиям с вопросами;
  • выражением.

В решении таких задач главное:

  • найти главное и сделать краткую запись;
  • разложить эту задачу на несколько простых и составить план решения;
  • помнить главное: по двум данным находим третье.

3. Задачи на понимание смысла действий умножения и деления

  • Важно запомнить названия компонентов действий и понять их смысл:

1-й множитель х 2-й множитель = произведение
делимое : делитель =частное

  • Ребенок должен понимать, что 1-й множитель показывает, КАКОЕ число повторяется а 2-й множитель показывает — СКОЛЬКО РАЗ оно повторяется.

Это очень важно для правильной записи в задачах, иначе получится бессмыслица.

Советы о том, как научить ребенка осознанно относиться к умножению и делению, вы найдете в нашей статье Как научить детей быстро считать: математика до школы. Если возникли проблемы с решением задач на умножение — сдайте чуть-чуть назад, закрепите осознание этого арифметического действия.

4. Простые задачи на умножение и деление

  • Очень важно понять и запомнить разницу «в «, «на».

«Во сколько раз» или «на сколько»?  Предлог «на» — это сложение или вычитание, а «в» — умножение или деление.

  • Важно понять и запомнить: чтобы узнать, во сколько раз одно число больше или меньше другого, нужно большее число разделить на меньшее.

 5. Составные задачи на все 4 арифметические действия

6. Задачи на цену, количество, стоимость

7. Задачи на движение

Это отдельная обширная тема, вернемся к ней позже.

Типичные ошибки в решении задач

Ошибка №1. Ребенок невнимательно прочитал условие задачи.

Часто бывает так, что ошибки возникают от невнимательности.  Так часто бывает в задачах с косвенным вопросом. Ребенок смотрит на цифры, вроде все логично, но… не верно.

Например: «У Маши 8 конфет, это на 2 меньше, чем у Кати. Сколько конфет у Кати».

Ребенок видит «на 2 меньше» и делает «логичный» вывод, что надо отнять. Отнять можно от бОльшего числа, т.е. сразу напрашивается решение 8-2=6. И ответ: 6 конфет у Кати. А ответ-то не тот! Если внимательно почитать условие, то станет понятно, что у Кати конфет больше чем у Маши. И вовсе тут не отнимать надо.

Как исправить ошибку. Сразу разберитесь с условием, поможет краткая запись.

Ошибка №2. Ребенок допустил ошибку в решении.

Когда в задаче несколько неизвестных, решение затрудняется, требуется выполнить не одно действие, а придумать целую цепочку рассуждений. 

Как исправить ошибку. Для начала определим, каких данных нам не хватает. Решаем по действиям. Находим нужные числа (помним правило: по двум неизвестным находим третье), подставляем их и отвечаем на вопрос задачи.

Ошибка №3. Неправильная запись ответа.

Часто ребенок пишет не то пояснение.

Как исправить ошибку.  Нужно внимательно прочитать вопрос задачи. Уяснить раз и навсегда, что ответ начинается с числа, а дальше пишем, что требовалось найти (переписываем формулировку вопроса задачи). 

Творческий подход в решении задач

www.craftykidsathome.com

  • Учите ребенка рассуждать.
  • Придумывайте задачи с лишними или недостающими данными.

Пусть ребенок сам вычеркнет лишнее, те данные, которые не влияют на решение.

  • Дайте условие, а ребенок пусть сам придумает ответ.
  • Пусть ребенок сам составит обратную задачу.
  • Придумать несколько задач на одно решение.
  • Придумать, как решить задачу другим способом и объяснить его.

На школу надейся, а сам не плошай

Заглянем в педагогику и «расшифруем» мысли умных и заслуженных, исходя из сегодняшних реалий.

В далеком 1867 году К. Ушинский сказал: «У хороших преподавателей дело выходит так, что арифметическая задача есть вместе занимательный рассказ, урок сельского хозяйства или домашней экономии, или историческая или статистическая тема и упражнение в языке».

«Расшифровка» следующая.

  • Ученика нужно поставить в такие условия, чтобы он оказался в эпицентре событий, т.е., решая задачу, видел ее применение в жизни.

Не всегда задачи в школьном учебнике «вдохновляют» современных школьников. Многим не ясно условие по одной простой причине: ребенок не имеет представления о том, что говорится. Например, задача про надои и бидоны с молоком, а городской «деть» и корову-то в глаза не видел, не то, что тонны молока в бидонах. Или в задаче использованы такие значения, которые в жизни нереальны — это затрудняет восприятие, т.к. ребенок все воспринимает буквально.

Задача родителей — помочь ребенку ПОНЯТЬ условие. Любым способом: хоть рисуй, хоть танцуй.

  • К решению задач нужно подходить творчески.

Интерес заставляет ребенка быть активным, а активность в свою очередь усиливает внимание.

В каждодневной жизни нам то и дело приходится решать задачи. Привлекайте ребенка, задавайте вопросы, просите совета. Например, тема ремонта. Вычислить метраж комнаты; просчитать нужное количество краски, зная расход на метр квадратный; купить линолеум, зная длину и ширину комнаты; просчитать, какой метраж выгоднее, если есть напольное покрытие шириной 2, 5 метра и 3 метра, чтобы меньше остатков было и по цене вышло выгоднее. Купить ткань на пошив постельного белья, зная размеры матраса. Примеров масса! И это работает гораздо эффективнее, чем «бездушная» задача в учебнике, которая совершенно не привязана к жизни и не вызывает эмоциональный отклик.

  • При решении жизненных задач у ребенка помимо всего прочего развивается наблюдательность, речь, появляется рабочее настроение, развиваются творческие способности и самостоятельность.

Через некоторое время вы заметите, что ребенок различными способами комбинирует информацию, с легкостью составляет задачи сам, находя идеи в окружающем мире, а не высасывая из пальца.

  • Когда ребенка просят составить собственную задачу, нужно следить и за содержанием, и за решением. Задача должна быть осмысленной и целесообразной.

Например, нельзя допускать таких «ляпов», как «Я съел 13 желтых груш и 20 зеленых яблок. Сколько фруктов я съел?» Задача теряет смысл, если она оторвана от жизни.

  • От задачи надо идти к примеру, а не наоборот.

Дети мыслят не абстрактно, а конкретными образами. Пример 12-6 ни о чем не говорит, а вот ситуация, когда из 12 человек 6 уже купили билеты на футбольный матч — это совсем другое дело. Тут ребенок не задумываясь ответит, что оставшиеся шестеро очень рискуют, нужно поторопиться, иначе билетов может не хватить и придется сидеть у телевизора, вместо того, чтобы активно скандировать на трибунах в поддержку любимой команды.

Лебединцев в своей книге «Введение в современную методику математики» писал: «То влияние, которое может оказывать обучение счислению и вообще математике на умственное развитие детей, находится в прямой зависимости от материала, которым мы пользуемся при обучении; если в учебном материале будут преобладать отвлеченные упражнения в действиях и хитроумные задачи с условиями, лишенными внутренней связи и, по существу, далекими от жизни, то, упражняя учащихся на таком материале, мы, может быть, и выработаем у них формальные навыки в вычислениях и, пожалуй, изощрим их ум для разгадывания разных ребусов и головоломок, но отнюдь не сделаем их более способными к правильному мышлению в жизни или какой-либо области знания…».

Французский педагог Жан Мосе тоже был уверен, что «заставлять ребенка начинать с отвлеченного правила и затем предлагать ему задачи — это значит идти наперекор ходу развития человеческого ума…».

Практические советы по решению задач от реальных мам

fb.ru

Что нам Ушинский, Лебединцев и Мосе, спросим у тех, кто «из нашей песочницы». Как они помогают своим детям решать задачи по математике, что «работает», какие приемы на практике доказали свою эффективность и помогли повысить успеваемость.

Татьяна, мама учеников 4 кл. и 6 кл. 

«Я знаю, что особую сложность у детей вызывают задачи на скорость, поэтому начала готовить своих мальчишек к этому уже с 1 класса. Когда ехали к бабушке в Пинск, говорили о скорости, засекали время, считали сколько мы проехали км, смотрели на знаки и вычисляли сколько нам останется времени, если мы будем ехать с такой же скоростью и сколько, если папа будет ехать с другой. В общем, я очень удивлялась, когда мои пацаны на скорость задачи решали как орехи. Я поняла, что в моем детстве не хватало практического представления того, о чем говорилось в задачах».

Ольга, мама ученика 1 кл. и ученицы 4 кл.

«С задачами старшая плохо дружит))  Почти всегда приходит за помощью. Стараюсь выработать алгоритм решения, но частенько упираюсь в «лень подумать». Если совсем «затык», рисуем схемы. На дополнительные задачи совсем нет времени, а сама по своей воле заниматься ими дочь точно не будет)) Иногда встречаются задачи с некорректно поставленным вопросом, тут приходится помогать с формулировкой ответа.

Младшего усадить за математику очень сложно. В те редкие моменты, когда дело доходит до задач, он их решает в уме и выдает ответ устно).»

Вероника, мама учеников 2 кл. и 4 кл.

«Младший задачи решает без проблем, но ненавидит чертить схемы к ним и писать пояснения. Старший ходит на факультатив по математике, дома домашку сам делает». 

Катерина, мама ученика 2 кл. и ученицы 5 кл.

«Сын отлично справляется сам. Он такие схемы рисует, что я иногда в шоке)). Если за помощью обращается дочь, стараюсь упростить условие задачи до понятных образов, а потом она сама догадывается, как сложную модель решить».

Татьяна, мама ученицы 5 кл.

«Чаще всего прибегаем к рисованию. Прямо вот как по условию… садимся и рисуем, как есть. Так сказать, наглядность помогает. Велосипедист выехал… значит рисуем человечка на велосипеде, город из которого он выехал и тд)))) Если катер плывет по течению, рисуем море, волны)))))) С пояснениями никогда исправлений со стороны учителя не было, да и у нас, собственно, тоже вопросов не возникало. Смотри по условию, что спрашивают — и пиши ответы возле каждого действия».

Наталья, мама ученика 5 кл.

«Приходилось объяснять дроби на примере сломанных карандашей, порванных в клочья бумажек. В гостях в тот момент был друг-проектировщик, он именно так решил наглядно пояснить сыну задачу. Я обычно прибегаю к помощи рисования. В задачах на скорость/время/расстояние рисовали целые истории: кто куда и на чем поехал, кого встретил по дороге и в какой момент. Порой решение задач превращалось в мультфильм, одного черновика обычно мало. Несколько раз решали задачи всей семьей: мама отдельно от папы, потом сравнивали результаты и каждый объяснял ребенку свой «самый рациональный и простой» способ. Как правило, у мужчин своя логика)), мое решение обычно отличается от папиного».

Уважаемые читатели! Делитесь в комментариях своими находками и сложностями в решении задач по математике с детьми. будем разы разобраться вместе и помочь советами и полезными статьями на интересующие вас темы. 

15 упражнений для тренировки мозга

Программистам без логики никуда. Поэтому время прокачать мозг: проверьте свои способности. Вам под силу эти логические задачи?

Полезно решать и логические задачи, и математические. Так вы развиваете логику и тренируете мозг. В силу профессии айтишнику крайне важно следить за тонусом своей главной «мышцы». Мозг любит задачки и головоломки, а ещё переключение внимания и отдых от рутины. Поэтому скорее приступим к развлечениям с пользой!

Логические задачи для разминки

1 задача

Поставьте правильное число вместо вопросительного знака:

4  5  6  7  8  9 
61 52 63 94 46 ?

Ответ

Числа нижнего ряда – квадраты чисел верхнего ряда с перестановкой цифр. Вместо знака ставьте число 18.

2 задача

Один парень в компании предложил друзьям такой спор:

– Спорим, я выставлю бутылку на середину комнаты и вползу в неё.

И получилось. Он победил.

Как парню посчастливилось это сделать?

Ответ

Он без труда вполз в неё – в комнату.

3 задача

Представьте ряд из шести чашек на столе. Три первые из них ничем не наполнены, а три следующие – с водой. Как добиться чередования пустых чашек и чашек с водой? Касаться разрешается только одной чашки. При этом толкать чашку чашкой запрещается.

Что вы предпримете?

Ответ

Возьмите пятую чашку, перелейте из неё воду во вторую и поставьте чашку на место.

4 задача

В санатории на лужайке двое мужчин заняты настольным теннисом. Один ударяет ракеткой так сильно, что теннисный шарик улетает далеко и попадает в трубу из стали. Труба зарыта в землю вертикально на три метра. Шарик лежит на дне трубы, то есть на расстоянии трёх метров от плоскости земли. У игроков нет другого шарика.

Ответьте, как спортсменам достать игральный шар без извлечения трёхметровой трубы из-под земли?

Ответ

Спортсмены наполнят трубу водой до краёв, и тогда шарик всплывёт.

5 задача

Получится ли у вас записать число 1000 с использованием только восьми восьмёрок и символов математического сложения?

Ответ

888 + 88 + 8 + 8 + 8 = 1000

Логические задачи основного комплекса

6 задача

Попробуйте установить принцип построения указанной последовательности:

8 2 9 0 1 5 7 3 4 6

Ответ

Последовательность цифр построена на основании алфавитного порядка их названий (восемь, два, девять, ноль и т. д.).

7 задача

Вообразите десятикилометровый мост через пролив. Максимальная нагрузка для него – 25 тонн. С начала этого моста стартовал грузовик, масса которого – ровно 25 тонн. Автомобиль продолжает движение к противоположному краю. Баланс моста пока не нарушен. Неожиданно, когда грузовик достиг середины этого путепровода, на него сел воробей со своим весом.

Вопрос к вам: приведёт ли вес птицы к нарушению балансировки и разрушению моста?

Ответ

Не приведёт. Так как грузовик преодолел путь до середины моста, что равно 5 км, расход потраченного топлива в разы превысил вес птицы.

8 задача

В одно и то же время к водному каналу приблизилось двое. Добраться до другого берега поможет лодка, которая рассчитана только на одну персону. Тем не менее, без стороннего участия оба переправились на противоположный берег на этой лодке.

Как люди справились?

Ответ

Люди приблизились к противоположным берегам водного канала.

9 задача

Отважного воина захватили слуги султана. Султан приказал отправить смельчака в темницу, где вместе с ним беспрерывно будут рядом два надзирателя. Один стражник исключительно правдив в разговоре, а второй лжёт. Внутри темницы оказались два закрытых входа. Войдёшь в один – выберешь «неволю навсегда», а в другой – «освобождение». Султан предложил воину избрать один вход: если это окажется «освобождение», то пленник свободен.

Воину позволили спросить надзирателей только один раз. То есть один вопрос на двоих надзирателей. Пленник не в курсе, кто из них лжец, а кто правдив. Надзиратели, несомненно, знают, какой вход подарит воину освобождение.

Какой вопрос задаст воин одному из стражников для достоверного определения входа, который означает «освобождение»?

Ответ

Воин задаст только один вопрос любому из надзирателей: «Если попросить твоего коллегу указать на вход «освобождение», то куда он направит меня?» В обеих ситуациях надзиратель укажет на «вход в неволю».

10 задача

В лесу десять родников с мёртвой водой: от первого до десятого. Мёртвая вода из родников с первого по девятый доступна каждому, а десятый родник во власти Кощея в пещере, в которую он никого не впускает. По вкусу и цвету мёртвую воду не отличить от обыкновенной, однако глоток воды из родника означает смерть. Спасёт только вода из родника с номером выше. Поэтому того, кто изначально выпьет десятую воду, ничего не спасёт.

Иванушка бросил вызов Кощею. Условились принести с собой чашку с водой и дать её осушить сопернику. Радости Кощея не было предела: «Ура! Я налью десятый яд, и Иванушка не спасётся! В то же время выпью то, что даст Иванушка, следом выпью десятый яд и останусь живым!»

В оговоренный день соперники столкнулись. Последовал честный обмен чашками. Содержимое выпито. При этом Кощей погиб, а Иванушка выжил.

Как Иванушка победил Кощея?

Ответ

Иванушка предложил Кощею обыкновенную воду. А Кощей, по неверным предположениям, «запил» её собственным десятым ядом. Перед встречей с соперником Иванушка выпил воду из любого родника. Вышло, что он запил яд Кощеевым десятым, что привело к нейтрализации яда.

Логические задачи для «растяжки»

11 задача

Отец решил задать своему сыну-школьнику каверзный вопрос: назови самое большое число. Ответ сына ошеломил отца, возразить было нечего.

Для программиста это дело лёгкое, правда? Но что сказал школьник?

Ответ

Тридцать первое. Предполагается число месяца.

12 задача

Воинственное племя захватило странника. Вождь хотел смерти страннику и позволил ему выбирать. Страннику разрешалось озвучить одну фразу. При правдивости фразы его сбросят с отвесной скалы. Окажись фраза лживая, и его отдадут львам на растерзание. Но странник подобрал такую фразу, которая подарила ему свободу.

Отгадайте, что это за фраза?

Ответ

Фраза: «Меня растерзают львы». Тогда, если бы вождь отдал странника львам на растерзание, то сказанная фраза стала бы правдивой, и его полагалось бы бросить с отвесной скалы. Но если странника сбросят со скалы, то фраза окажется лживой. Вождь посчитал, что исключительно правильным исходом будет подарить свободу страннику.

13 задача

После гулянки мужчина направлялся домой слегка навеселе. Он шёл по центру песчаной проселочной дороги. Путь не освещался лунным светом. К тому же, на дороге отсутствовали фонари. Одежда мужчины была чёрной. Вдруг на дорогу выехал автомобиль с выключенными фарами. В конце концов, водитель заметил мужчину и свернул.

Как ему удалось увидеть пешехода?

Ответ

На улице стоял день.

Логические задачи для заминки

14 задача

Вам даётся три письма. Одно придётся незамедлительно проглотить. В каждом письме найдёте пару предложений. Два предложения в одном письме истинные, в другом – ложные, а в третьем – пополам – истинное и ложное. Смотрите, какие там предложения:

Первое письмо:

  1. Не ешьте это письмо.
  2. Непременно съешьте второе письмо.

Второе письмо:

  1. Не стоит есть первое письмо.
  2. Жуйте третье письмо.

Третье письмо:

  1. Есть это письмо не стоит.
  2. Скорее съедайте первое письмо.

Поделитесь своим мнением, какое письмо съесть?

Ответ

Третье письмо.

15 задача

Вы очутились в помещении, в котором четыре двери и крошечное окошко. Три двери фальшивые, то есть за ними сразу кирпичная кладка. И одна дверь с выходом на улицу. Вам дали ключ, который открывает все четыре двери, однако вы без понятия, какая дверь выведет на улицу. Попытаться можно один раз. При открывании одной двери оставшиеся замки блокируются механически и безвозвратно. Вдобавок комната тёмная и слегка озаряется светом одной свечи.

Какие вы примете меры, чтобы отыскать единственную дверь, которая ведёт на улицу?

Ответ

Стоит распахнуть окошко и подставлять свечу по очереди к дверям: к щелям или к замочной скважине. При этом внимательно смотреть на пламя свечи. Колебание пламени будет указывать на выход.

Эти логические задачи показались вам лёгкими или не очень?

Попробуйте другие логические задачи:

Math.com — Репетиторство по математике

Чтобы улучшить процесс обучения математике у вас или вашего ребенка, было бы разумно подумать о сохранении этих услуг.
репетитора по математике. При выборе репетитора следует учитывать несколько факторов:

Хороший репетитор по математике должен хорошо владеть материалом, который он преподает.Таким образом, это
Важно, чтобы они имели сильное формальное образование в области математики. При обучении маленьких детей завершение
по крайней мере, будет рекомендован аттестат об окончании средней школы. Обучение старшеклассников, в старших классах школы или колледжа,
идеальный репетитор должен иметь как минимум степень бакалавра с упором на математику (инженерное дело или
к научным степеням также часто предъявляются серьезные требования по математике).

Чтобы помочь другим учиться, необходимы отличные коммуникативные навыки и способность определять лучшие
способ передать материал таким образом, чтобы он соответствовал индивидуальным потребностям учащегося.Предыдущий опыт
в репетиторстве или преподавании настоятельно рекомендуется при поиске репетитора по математике, и что
в идеале опыт должен включать уровень класса материала, который вы изучаете. Лучшее репетиторство
компании инвестируют ресурсы в постоянное обучение своих сотрудников новейшим методам обучения, чтобы
помочь учащимся усвоить материал.

Расположение и наличие

Чтобы свести к минимуму время и командировочные расходы, потраченные на дорогу, репетиторство по математике идеально подойдет к вам.А
у большой компании, занимающейся репетиторством по математике с большим количеством заведений, больше шансов найти место рядом с вами. Некоторое обучение
компании проводят обучение по телефону или онлайн, либо в качестве основного метода общения, либо в качестве
дополнение к традиционным классным комнатам.

Некоторые репетиторы доступны только в заранее запланированное время, в то время как с другими можно связаться в другое время.
часов, а в некоторых случаях даже круглосуточно (например, некоторые онлайн-сервисы). В зависимости от ваших потребностей,
это может быть важно, а может и нет.

Многие академические исследования документально подтверждают, что учащиеся лучше всего учатся, когда им уделяют больше внимания.
к их индивидуальным потребностям. Таким образом, рекомендуется использовать классы меньшего размера или даже индивидуальное обучение.
чтобы получить максимальную пользу от репетиторства по математике. Обычно особенно полезны самые маленькие дети.
от небольших классных комнат, но это также может относиться и к старшим.

Лучший способ обучения — это практика, практика и еще раз практика.Это можно сделать
через домашние задания или практические занятия в классе. Регулярное тестирование важно для
убедитесь, что материал усваивается. Компании, занимающиеся репетиторством по математике, часто помогают
для подготовки к конкретным стандартизированным тестам (например, SAT, GRE, GMAT, LSAT), чтобы иметь доступ к
тесты в том же формате, что и ожидаемые стандартизированные экзамены.

Репетиторам нужно платить за свое время (у них есть семьи, которые нужно кормить),
так что ожидать хороших репетиторов по математике бесплатно — нереально.Чтобы получить классные комнаты меньшего размера или индивидуальные
Репетиторство, естественно, будет стоить немного дороже. Является ли это хорошей инвестицией
будет зависеть от индивидуальных потребностей учащегося и степени его энтузиазма
преуспеть в математике. Некоторые компании, занимающиеся репетиторством по математике, предлагают финансовую помощь или скидки, поэтому
не помешает спросить, доступны ли они (скидка на лучшее обучение по математике
компания может сделать его более выгодным, чем полная цена в меньшей компании).Образовательные
расходы могут быть вычтены из вашего подоходного налога, в зависимости от того, где вы живете,
поэтому не забудьте сохранить квитанции и спросить у своего бухгалтера дополнительную информацию.

[Вернуться к началу]

7 простых математических уравнений, которые стали вирусными и разделили Интернет

Время от времени в Интернете публикуются математические задачи, которые становятся вирусными, во многом из-за того, что кажется, что никто не может прийти к согласию по ответу

Отлично умы сходятся, и все же их расчеты не дают одинаковых результатов.Вот реальные ответы на некоторые из уравнений, которые заставили интернет-пользователей коллективно ломать голову.

СВЯЗАННЫЕ: 10 НАИБОЛЕЕ ВАЖНЫХ УРАВНЕНИЙ В ИСТОРИИ

8 ÷ 2 (2 + 2) =?

Это ответ 16, или 1 ?

Уравнение стало вирусным после того, как пользователь Twitter @pjmdolI опубликовал его летом.

решения решают эту проблему pic.twitter.com/0RO5zTJjKk

— em ★ (@pjmdolI) 28 июля 2019 г.

Согласно Insider , те, кто получил 1 в качестве ответа, использовали устаревшую версию порядка операций .

Вот подробное описание из MindYourDecisions на YouTube.

9 — 3 ÷ 1/3 + 1 =?

В 2016 году тест показал, что только 60% японцев 20-летнего возраста смогли решить это уравнение, по сравнению с 90% в 80-х годах.

Опять же, проблема связана с порядком операций.

Самая распространенная ошибка, по словам Преша Talkwalkar из MindYourDecision и автора книги The Joy of Game Theory , — это написание 1/3 без скобок, когда люди используют калькулятор для ее решения.2 ÷ 2 (3) + 4 =?

Это 10 или 58 ?

Два разных способа вычисления ответа дают разные результаты.

Источник: (снимок экрана) MindYourDecisions / YouTube

И снова, как указывает GeniusInsomniac (видео ниже), мораль этой истории заключается в важности знания правильного порядка действий.

Это выглядит так: круглые скобки, показатели степени, умножение и / или деление (в зависимости от того, что наступит раньше), сложение и / или вычитание (в зависимости от того, что наступит раньше).

6-1 x 0 + 2 ÷ 2 =?

И снова Преш Талвалкар разрешает математические споры. Казалось бы, простая математическая задача вызвала разногласия по поводу того, будет ли ответ 7 или 1 .

Правильный ответ 7. Почему? Вы угадали: порядок действий.

60 ÷ 5 (7-5) =?

Талвалкар отмечает, что приведенное выше уравнение дает разные результаты в зависимости от того, какой калькулятор используется.

Однозначный ответ, по его словам, — 24 , основанный на современной интерпретации PEMDAS / BODMAS.

230-220 ÷ 2 =?

Конечно, ответы не всегда однозначны. Примерно так же, как этот язык можно интерпретировать по-разному, математические задачи тоже.

Вот почему кандидат математических наук. профессор и Преш Талвалкар из MindYourDecisions дали разные ответы на эту проблему. Талвалкар сказал 120 и доктор философии.Д., профессор сказал: 5 .

7 + 7 ÷ 7 + 7 x 7-7 =?

Еще один урок по порядку работы. Это уравнение действительно сводит его к основам.

Источник: (снимок экрана) MindYourDecisions / YouTube

Талвалкар подчеркивает важность аббревиатур PEMDAS или BODMAS для запоминания правильного порядка.

120 задач по математике для учащихся 1–8 классов

120 задач по математике для учащихся 1–8 классов | Prodigy Education Вы сидите за своим столом, готовые провести математическую викторину, тест или задание.Вопросы перетекают в документ, пока вы не дойдете до раздела, посвященного проблемам с текстом. Помог бы толчок творчества. Но этого не произошло. Независимо от того, являетесь ли вы учителем 3-го класса или учителем 8-го класса, готовящим учеников к старшей школе, воплощение математических концепций в примеры из реального мира, безусловно, может быть проблемой. Этот ресурс дает вам толчок к творчеству. Содержит примеры и шаблоны задач по математике для 1-8 классов. Всего 120 примеров. Помогая вам разобраться в них, чтобы найти вопросы для ваших учеников, ресурс разделен на категории по следующим навыкам с некоторым перекрытием между темами: Список примеров дополнен советами по созданию увлекательных и сложных математических словесных задач.

120 Математические задачи со словами, классифицированные по навыкам

Задачи со сложением слов

Подходит для: 1-й класс, 2-й класс 1. Добавляем к 10: Ариэль играл в баскетбол. 1 из ее выстрелов попал в обруч. 2 ее выстрела не попали в обруч. Сколько всего было выстрелов? 2. Добавление к 20: У Адрианны есть 10 кусочков жевательной резинки, которыми она может поделиться со своими друзьями. На всех ее подруг не хватило жевательной резинки, поэтому она пошла в магазин за еще тремя кусочками жевательной резинки.Сколько кусочков жевательной резинки сейчас у Адрианны? 3. Добавление к 100: У Адрианны есть 10 кусочков жевательной резинки, которыми она может поделиться с друзьями. На всех ее подруг не хватило жевательной резинки, поэтому она пошла в магазин и купила 70 кусочков клубничной жевательной резинки и 10 кусочков жевательной резинки. Сколько кусочков жевательной резинки сейчас у Адрианны? 4. Добавление Чуть больше 100: В ресторане 175 обычных стульев и 20 стульев для младенцев. Сколько всего стульев в ресторане? 5.Добавляем к 1000: Сколько печенья вы продали, если продали 320 шоколадных печений и 270 ванильных печений? 6. Добавление и более 10 000: Обычно магазин товаров для хобби продает 10 576 торговых карточек в месяц. В июне в магазине товаров для хобби было продано на 15 498 карточек больше, чем обычно. В целом, сколько коллекционных карточек было продано в магазине для хобби в июне? 7. Сложение 3 чисел: У Билли дома было 2 книги. Он пошел в библиотеку, чтобы достать еще 2 книги. Затем он купил 1 книгу.Сколько книг у Билли сейчас? 8. Добавление трех чисел к 100 и более: Эшли купила большой мешок конфет. В сумке было 102 синих конфеты, 100 красных и 94 зеленых. Сколько всего было конфет?

Задачи на вычитание слов

Подходит для: 1-й класс, второй класс 9. Вычитание до 10: Всего в пиццерии было 3 пиццы. Покупатель купил 1 пиццу. Сколько пиццы осталось? 10. Вычитаем до 20: Ваша подруга сказала, что у нее 11 наклеек.Когда вы помогли ей убрать стол, у нее было всего 10 наклеек. Сколько наклеек не хватает? 11. Вычитаем до 100: У Адрианны есть 100 кусочков жевательной резинки, которыми она может поделиться со своими друзьями. Когда она пошла в парк, она разделила 10 кусочков клубничной жевательной резинки. Когда она вышла из парка, Адрианна поделилась еще 10 кусочками жевательной резинки. Сколько кусочков жевательной резинки сейчас у Адрианны? 12. Вычитание Немного больше 100: Ваша команда набрала 123 очка.В первом тайме было набрано 67 очков. Сколько было забито во втором тайме? 13. Вычитаем до 1000: У Натана большая муравьиная ферма. Он решил продать несколько своих муравьев. Он начал с 965 муравьев. Продал 213. Сколько муравьев у него сейчас? 14. Вычитая до 10 000 и более: Обычно магазин товаров для хобби продает 10 576 торговых карточек в месяц. В июле в магазине товаров для хобби было продано 20 777 коллекционных карточек. Сколько коллекционных карточек было продано в магазине в июле по сравнению с обычным месяцем? 15.Вычитая 3 числа: У Шарлин была пачка из 35 карандашей. 6 она отдала своей подруге Терезе. Она отдала 3 своей подруге Мэнди. Сколько мелков осталось у Шарлин? 16. Вычитание трех чисел от 100 и более: Эшли купила большой мешок конфет, чтобы поделиться с друзьями. Всего конфет было 296 штук. Она подарила Мариссе 105 конфет. Еще она подарила Кайле 86 конфет. Сколько конфет осталось?

Задачи на умножение слов

Подходит для: 2-й класс, 3-й класс 17.Умножение однозначных целых чисел: Адрианне нужно разрезать сковороду с пирожными на кусочки. Она нарезает на сковороду 6 ровных столбиков и 3 ровных ряда. Сколько у нее пирожных? 18. Умножение 2-значных целых чисел: В кинотеатре 25 рядов сидений по 20 мест в каждом ряду. Сколько всего мест? 19. Умножение целых чисел, заканчивающееся на 0: Компания по производству одежды предлагает 4 различных вида толстовок. Ежегодно компания производит 60 000 толстовок каждого вида.Сколько свитшотов компания производит каждый год? 20. Умножение 3 целых чисел: Каменщик укладывает кирпичи в 2 ряда по 10 кирпичей в каждом ряду. Сверху каждого ряда находится стопка из 6 кирпичей. Сколько всего кирпичей? 21. Умножение 4 целых чисел: Кэли зарабатывает 5 долларов в час, разнося газеты. Она доставляет газеты 3 дня в неделю по 4 часа за раз. Сколько денег заработает Кэли после доставки газет в течение 8 недель?

Задачи с разделением слов

Подходит для: 3-й класс, 4-й класс, 5-й класс 22.Деление однозначных целых чисел: если у вас есть 4 конфеты, поровну разделенных на 2 пакета, сколько конфет находится в каждом пакете? 23. Разделение 2-значных целых чисел: Если у вас есть 80 билетов на ярмарку, и каждая поездка стоит 5 билетов, сколько поездок вы сможете совершить? 24. Разделительные числа, оканчивающиеся на 0: У школы есть 20 000 долларов на покупку нового компьютерного оборудования. Если каждая единица оборудования стоит 50 долларов, сколько всего ее может купить школа? 25.Деление 3 целых чисел: Мелисса покупает 2 упаковки теннисных мячей на общую сумму 12 долларов. Всего 6 теннисных мячей. Сколько стоит 1 упаковка теннисных мячей? Сколько стоит 1 теннисный мяч? 26. Переводчик: Итальянский ресторан получил партию из 86 телячьих котлет. Если на блюдо нужно 3 котлеты, сколько котлет останется в ресторане после приготовления как можно большего количества блюд?

Задачи со смешанными операциями со словами

Подходит для: 3-й класс, 4-й класс, 5-й класс 27.Смешивание сложения и вычитания: в библиотеке 235 книг. В понедельник вывозят 123 книги. Во вторник возвращено 56 книг. Сколько сейчас книг? 28. Смешивание, умножение и деление: Есть группа из 10 человек, которые заказывают пиццу. Если каждый человек получает 2 куска, а у каждой пиццы 4 куска, сколько пиццы им следует заказать? 29. Смешивание, умножение, сложение и вычитание: У Ланы 2 пакета по 2 шарика в каждом.У Маркуса 2 сумки по 3 шарика в каждой. Сколько еще шариков у Маркуса? 30. Смешивание, сложение и вычитание: У Ланы 3 мешка с одинаковым количеством шариков, всего 12 шариков. У Маркуса 3 сумки с таким же количеством шариков, всего 18 шариков. Сколько еще шариков у Маркуса в каждой сумке?

Упорядочивание слов и задачи со смыслом чисел

Подходит для: 2-й класс, 3-й класс 31. Подсчет для предварительного умножения: в вашем классе есть 2 классные доски.Если на каждую классную доску нужно 2 куска мела, сколько всего кусков вам нужно? 32. Подсчет перед предварительным просмотром: В вашем классе 3 классные доски. На каждой доске по 2 мелка. Это означает, что всего есть 6 мелков. Если вы уберете по 1 мелу с каждой доски, сколько всего их будет? 33. Составление чисел: Какое число 6 десятков и 10 единиц? 34. Угадайки: У меня семерка в разряде десятков.У меня четное число вместо единиц. Мне меньше 74. Какой я номер? 35. В поисках порядка: В хоккейной игре Митчелл набрал больше очков, чем Уильям, но меньше очков, чем Остон. Кто набрал больше всего очков? Кто набрал меньше всего очков?

Задачи со словами на дроби

Подходит для: 3-й класс, 4-й класс, 5-й класс, 6-й класс 36. Поиск фракций группы: Джулия пошла в 10 домов на своей улице на Хэллоуин. В 5 домах ей подарили плитку шоколада.В какой части домов на улице Джулии ей дали плитку шоколада? 37. Поиск фракций единицы: Хизер рисует портрет своей лучшей подруги Лизы. Чтобы было легче, она делит портрет на 6 равных частей. Какая дробь представляет каждую часть портрета? 38. Сложение дробей с одинаковыми знаменателями: Ной проходит ⅓ километра до школы каждый день. Он также проходит ⅓ километра, чтобы вернуться домой после школы. Сколько всего километров он проходит? 39.Вычитание дробей с одинаковыми знаменателями: На прошлой неделе Уитни подсчитала количество коробок сока, которые у нее были на школьные обеды. У нее было случая. На этой неделе осталось ⅕ случая. Сколько вина выпила Уитни? 40. Сложение целых чисел и дробей с одинаковыми знаменателями: В обеденное время в кафе-мороженом подавали 6 ложек шоколадного мороженого, 5 ложек ванили и 2 ложки клубники. Сколько всего шариков мороженого обслужили в салоне? 41.Вычитание целых чисел и дробей с одинаковыми знаменателями: На вечеринке у Хайме было 5 ⅓ бутылок колы, чтобы ее друзья выпили. Она сама выпила бутылки. Ее друзья выпили 3 ⅓. Сколько бутылок колы осталось у Хайме? 42. Сложение дробей с непохожими знаменателями: Кевин выполнил ½ задания в школе. Вернувшись в тот вечер домой, он выполнил ⅚ другого задания. Сколько заданий выполнил Кевин? 43. Вычитание дробей с непохожими знаменателями: Собирая школьные обеды для своих детей, Пэтти использовала упаковки ветчины.Еще она использовала ½ упаковки индейки. Насколько больше ветчины, чем индейки, использовала Пэтти? 44. Умножение дробей: Во время урока физкультуры в среду ученики пробежали километра. В четверг они пробежали ½ километра, как в среду. Сколько километров пробежали студенты в четверг? Запишите свой ответ дробью. 45. Разделение на фракции: Производитель одежды использует флакона цветного красителя для изготовления одной пары брюк. Производитель вчера использовал бутылки.Сколько пар брюк изготовил производитель? 46. Умножение дробей на целые числа: Марк на этой неделе выпил ⅚ пакета молока. Фрэнк выпил в 7 раз больше молока, чем Марк. Сколько пакетов молока выпил Фрэнк? Запишите свой ответ дробью, целым или смешанным числом.

Задачи с десятичными знаками

Подходит для: 4-й класс, 5-й класс 47. Добавление десятичных знаков: у вас в миске 2,6 грамма йогурта, и вы добавляете еще одну ложку 1,3 грамма.Сколько всего йогурта у вас есть? 48. Вычитание десятичных знаков: У Джеммы было 25,75 грамма глазури для приготовления торта. Она решила использовать только 15,5 грамма глазури. Сколько глазури осталось у Джеммы? 49. Умножение десятичных дробей на целые числа: Маршалл проходит в общей сложности 0,9 км до школы и обратно каждый день. Сколько километров он пройдет через 4 дня? 50. Разделение десятичных знаков на целые числа: Чтобы сделать Пизанскую башню из спагетти, миссис.Робинсон купил 2,5 килограмма спагетти. Всего ее ученики смогли построить 10 наклонных башен. Сколько килограммов спагетти нужно для изготовления 1 падающей башни? 51. Смешивание сложения и вычитания десятичных знаков: У Рокко в холодильнике 1,5 литра апельсиновой соды и 2,25 литра виноградной газировки. У Антонио 1,15 литра апельсиновой газировки и 0,62 литра виноградной газировки. Насколько больше газировки у Рокко, чем у Анджело? 52. Смешивание умножения и деления десятичных знаков: 4 дня в неделю Лаура занимается боевыми искусствами на 1 ед.5 часов. Учитывая, что в неделе 7 дней, каково ее среднее время занятий в день каждую неделю?

Сравнение и упорядочение словарных задач

Подходит для: Детский сад, 1-й класс, 2-й класс 53. Сравнение однозначных целых чисел: у вас 3 яблока, а у вашего друга 5 яблок. У кого больше? 54. Сравнение 2-значных целых чисел: У вас 50 конфет, а у вашего друга 75 конфет. У кого больше? 55. Сравнение различных переменных: На детской площадке есть 5 баскетбольных мячей.На детской площадке установлено 7 футбольных мячей. Есть еще баскетбольные мячи или футбольные мячи? 56. Последовательность 1-значных целых чисел: У Эрика 0 наклеек. Каждый день он получает еще 1 наклейку. Сколько дней до того, как он получит 3 наклейки? 57. Пропуск по нечетным числам: Натали начала с 5. Она прыгнула по пятеркам. Могла ли она сказать число 20? 58. Пропуск по четным числам: Наташа начала с 0. Она считала по восьмеркам. Могла ли она сказать число 36? 59.Последовательность 2-значных чисел: Каждый месяц Джереми добавляет такое же количество карточек в свою коллекцию бейсбольных карточек. В январе у него было 36. В феврале 48. 60 марта. Сколько бейсбольных карточек будет у Джереми в апреле?

Задачи по времени

Подходит для: 1-й класс, 2-й класс 66. Преобразование часов в минуты: Джереми помогал своей маме 1 час. Сколько минут он ей помогал? 69. Время добавления: Если вы просыпаетесь в 7:00 утра и вам требуется 1 час 30 минут, чтобы собраться и пойти в школу, в какое время вы придете в школу? 70.Время вычитания: Если поезд отправляется в 14:00. и прибывает в 16:00, сколько времени пассажиры находились в поезде? 71. Определение времени начала и окончания: Ребекка вышла из магазина своего отца, чтобы пойти домой в двадцать семь вечера. Через сорок минут она была дома. Во сколько она приехала домой?

Задачи со словами о деньгах

Подходит для: 1-й класс, 2-й класс, 3-й класс, 4-й класс, 5-й класс 60. Добавление денег: Томас и Мэтью копят деньги, чтобы вместе купить видеоигру.Томас сэкономил 30 долларов. Мэтью сэкономил 35 долларов. Сколько денег они накопили в общей сложности? 61. Вычитание денег: Томас накопил 80 долларов. На свои деньги он покупает видеоигру. Видеоигра стоит 67 долларов. Сколько денег у него осталось? 62. Умножение денег: Тим получает 5 долларов за доставку бумаги. Сколько у него будет денег после 3-х раздачи бумаги? 63. Разделение денег: Роберт потратил 184,59 доллара на покупку трех хоккейных клюшек.Если каждая хоккейная клюшка была одинаковой по цене, сколько стоила 1? 64. Сложение денег с десятичными знаками: Вы пошли в магазин и купили жевательную резинку за 1,25 доллара и присоску за 0,50 доллара. Сколько было у вас всего? 65. Вычитание денег с десятичными знаками: Вы пошли в магазин с 5,50 долларами. Вы купили жевательную резинку за 1,25 доллара, плитку шоколада за 1,15 доллара и присоску за 0,50 доллара. Сколько у тебя осталось денег? 67. Применение пропорциональных отношений к деньгам: Якоб хочет пригласить 20 друзей на свой день рождения, который обойдется его родителям в 250 долларов.Если он вместо этого решит пригласить 15 друзей, сколько денег это будет стоить его родителям? Предположим, что отношение прямо пропорционально. 68. Применение процентов к деньгам: Ретта положил 100 долларов США на банковский счет, который приносит 20% годовых. Сколько процентов будет накоплено за 1 год? И если она не снимает деньги, сколько денег будет на счету через 1 год?

Проблемы физического измерения со словами

Подходит для: 1-й, 2-й, 3-й, 4-й класс 72.Сравнение размеров: линейка Кассандры 22 сантиметра в длину. Линейка апреля имеет длину 30 сантиметров. На сколько сантиметров длиннее линейка апреля? 73. Измерения в контексте: Представьте школьный автобус. Какая единица измерения лучше всего описывает длину автобуса? Сантиметры, метры или километры? 74. Добавление измерений: Папа Миши хочет сэкономить на бензине, поэтому он отслеживает, сколько он потребляет. В прошлом году папа Миши использовал 100 литров бензина.В этом году ее отец использовал 90 литров бензина. Сколько всего газа он использовал за два года? 75. Вычитание измерений: Папа Миши хочет сэкономить на бензине, поэтому он отслеживает, сколько он потребляет. За последние два года папа Миши использовал 200 литров бензина. В этом году он использовал 100 литров газа. Сколько газа он использовал в прошлом году? 76. Умножение объема и массы: Кира хочет убедиться, что у нее крепкие кости, поэтому она выпивает 2 литра молока каждую неделю.Сколько литров молока выпьет Кира через 3 недели? 77. Разделение объема и массы: Лилиан занимается садоводством, поэтому она купила 1 килограмм земли. Она хочет равномерно распределить почву между двумя растениями. Сколько получит каждое растение? 78. Преобразование массы: Ингер идет в продуктовый магазин и покупает 3 тыквы, каждая из которых весит 500 граммов. Сколько килограммов кабачков купила Ингер? 79. Конвертируемый объем: У Шэда есть киоск для лимонада, и он продал 20 чашек лимонада.Каждая чашка была 500 миллилитров. Сколько литров всего продала Шэд? 80. Длина преобразования: Стейси и Мильда сравнивают свой рост. Рост Стейси 1,5 метра. Милда на 10 сантиметров выше Стейси. Какой рост у Милды в сантиметрах? 81. Расстояние и направление: Автобус отправляется из школы, чтобы отвезти учеников на экскурсию. Автобус едет на 10 километров на юг, 10 километров на запад, еще 5 километров на юг и 15 километров на север.В каком направлении должен ехать автобус, чтобы вернуться в школу? Сколько километров он должен пройти в этом направлении?

Соотношение и процентное соотношение словарных задач

Подходит для: 4-й класс, 5-й класс, 6-й класс 82. Нахождение недостающего числа: Соотношение трофеев Дженни и трофеев Мередит составляет 7: 4. У Дженни 28 трофеев. Сколько у Мередит? 83. Поиск недостающих номеров: Соотношение трофеев Дженни и трофеев Мередит составляет 7: 4.Разница между числами — 12. Какие числа? 84. Сравнение коэффициентов: В младшем школьном оркестре работают 10 саксофонистов и 20 трубачей. В старшем оркестре школы 18 саксофонистов и 29 трубачей. У какого оркестра более высокое соотношение трубачей и саксофонистов? 85. Определение процентного соотношения: Мэри опросила учеников своей школы, чтобы определить их любимые виды спорта. 455 из 1200 студентов назвали хоккей своим любимым видом спорта.Какой процент студентов назвал хоккей своим любимым видом спорта? 86. Определение процента изменения: Десять лет назад население Оквилла составляло 67 624 человека. Теперь он на 190% больше. Каково население Оквилля в настоящее время? 87. Определение процентов чисел: В пункте проката коньков 60% из 120 коньков предназначены для мальчиков. Если остальные коньки для девочек, сколько их? 88. Расчет средних значений: В течение 4 недель Уильям был помощником на уроках плавания.Первую неделю он работал волонтером по 8 часов. Он работал волонтером 12 часов на второй неделе и еще 12 часов на третьей неделе. На четвертой неделе он работал волонтером по 9 часов. Сколько часов в среднем он работал волонтером в неделю?

Вероятность и проблемы со связью данных в словах

Подходит для: 4-й класс, 5-й класс, 6-й класс, 7-й класс 89. Понимание предпосылки вероятности: Джон хочет узнать любимое телешоу своего класса, поэтому он опрашивает всех мальчиков.Будет ли выборка репрезентативной или необъективной? 90. Понимание материальной вероятности: Грани на большом количестве кубиков помечены цифрами 1, 2, 3, 4, 5 и 6. Вы бросаете кубик 12 раз. Сколько раз вы должны ожидать, что вам выпадет 1? 91. Изучение дополнительных событий: Цифры от 1 до 50 находятся в шляпе. Если вероятность выпадения четного числа составляет 25/50, какова вероятность НЕ выпадать четное число? Выразите эту вероятность дробью. 92.Исследование экспериментальной вероятности: Пиццерия недавно продала 15 пицц. 5 из этих пицц были пепперони. Отвечая дробью, какова экспериментальная вероятность того, что следующая пицца будет пепперони? 93. Знакомство с взаимосвязями данных: Маурита и Феличе проходят по 4 теста. Вот результаты 4 тестов Мауриты: 4, 4, 4, 4. Вот результаты 3 из 4 тестов Феличе: 3, 3, 3. Если среднее значение Мауриты по 4 тестам на 1 балл выше, чем у Феличе, каков результат? оценка 4-го теста Феличе? 94.Представляем пропорциональные отношения: Store A продает 7 фунтов бананов за 7 долларов. Магазин B продает 3 фунта бананов по цене 6 долларов. В каком магазине выгоднее? 95. Написание уравнений для пропорциональных отношений: Лайонел любит футбол, но у него проблемы с мотивацией к тренировкам. Итак, он стимулирует себя с помощью видеоигр. Существует пропорциональная зависимость между количеством упражнений, которые выполняет Лайонел, в x , и тем, сколько часов он играет в видеоигры, в y .Когда Лайонел выполняет 10 упражнений, он 30 минут играет в видеоигры. Напишите уравнение отношения между x и y .

Геометрические задачи со словом

Подходит для: 4-й класс, 5-й класс, 6-й класс, 7-й класс, 8-й класс 96. Представляем периметр: в театре 4 стула в ряд. Всего 5 рядов. Если использовать строки в качестве единицы измерения, каков периметр? 97. Зона представления: В театре 4 стула в ряд.Всего 5 рядов. Сколько всего стульев? 98. Введение Том: Аарон хочет знать, сколько конфет может вместить его контейнер. Контейнер имеет высоту 20 сантиметров, длину 10 сантиметров и ширину 10 сантиметров. Каков объем контейнера? 99. Понимание 2D-форм: Кевин рисует фигуру с 4 равными сторонами. Какую форму он нарисовал? 100. Обнаружение периметра 2D-форм: Митчелл написал свои домашние задания на листе квадратной бумаги.Каждая сторона бумаги по 8 сантиметров. Какой периметр? 101. Определение площади 2D-форм: Одна торговая карточка имеет длину 9 см и ширину 6 см. Какая у него площадь? 102. Что такое 3D-фигуры: Марта рисует фигуру с 6 квадратными гранями. Какую форму она нарисовала? 103. Определение площади поверхности трехмерных фигур: Какова площадь поверхности куба шириной 2 см, высотой 2 см и длиной 2 см? 104.Определение объема трехмерных фигур: Контейнер для конфет Аарона имеет высоту 20 сантиметров, длину 10 сантиметров и ширину 10 сантиметров. Контейнер Брюса имеет высоту 25 сантиметров, длину 9 сантиметров и ширину 9 сантиметров. Найдите объем каждого контейнера. В зависимости от объема, чей контейнер может вместить больше конфет? 105. Определение прямоугольных треугольников: Треугольник имеет следующие длины сторон: 3 см, 4 см и 5 см. Этот треугольник прямоугольный? 106.Определение равносторонних треугольников: Треугольник имеет следующие длины сторон: 4 см, 4 см и 4 см. Что это за треугольник? 107. Определение равнобедренных треугольников: Треугольник имеет следующие длины сторон: 4 см, 5 см и 5 см. Что это за треугольник? 108. Определение треугольников из чешуи: Треугольник имеет следующие длины сторон: 4 см, 5 см и 6 см. Что это за треугольник? 109. Обнаружение периметра треугольников: Луиджи построил палатку в форме равностороннего треугольника.Периметр 21 метр. Какова длина каждой стороны палатки? 110. Определение площади треугольников: Какова площадь треугольника с основанием в 2 единицы и высотой 3 единицы? 111. Применение теоремы Пифагора: Прямоугольный треугольник имеет длину одной стороны без гипотенузы 3 дюйма и длину гипотенузы 5 дюймов. Какова длина другой стороны без гипотенузы? 112. Определение диаметра круга: Жасмин купила новый круглый рюкзак.Его площадь составляет 370 квадратных сантиметров. Какой диаметр у круглого рюкзака? 113. В поисках области круга: Круглый щит Капитана Америки имеет диаметр 76,2 сантиметра. Какова площадь его щита? 114. В поисках радиуса круга: Скайлар живет на ферме, где его отец держит круглый кукурузный лабиринт. Кукурузный лабиринт имеет диаметр 2 километра. Каков радиус лабиринта?

Проблемы с переменными словами

Подходит для: 6 класс, 7 класс, 8 класс 115.Определение независимых и зависимых переменных: Виктория печет кексы для своего класса. Количество кексов, которые она готовит, зависит от того, сколько у нее одноклассников. Для этого уравнения м — количество кексов, а c — количество одноклассников. Какая переменная является независимой, а какая зависимой? 116. Написание переменных для сложения: В прошлом футбольном сезоне Триш забила г голов. Алекса забила на 4 гола больше, чем Триш.Напишите выражение, показывающее, сколько голов забила Алекса. 117. Написание выражений переменных для вычитания: Элизабет ест здоровый, сбалансированный завтрак b раз в неделю. Мэдисон иногда пропускает завтрак. В общей сложности Мэдисон съедает на 3 завтрака меньше в неделю, чем Элизабет. Напишите выражение, показывающее, сколько раз в неделю Мэдисон завтракает. 118. Написание переменных выражений для умножения: В прошлом хоккейном сезоне Джек забил г голов.Патрик забил вдвое больше голов, чем Джек. Напишите выражения, показывающие, сколько голов забил Патрик. 119. Написание выражений переменных для Division: У Аманды c плиток шоколада. Она хочет равномерно распределить плитки шоколада между 3 друзьями. Напишите выражение, показывающее, сколько плиток шоколада получит один из ее друзей. 120. Решение уравнений с двумя переменными: Это уравнение показывает, как сумма, которую Лукас зарабатывает на внешкольной работе, зависит от того, сколько часов он работает: e = 12h .Переменная ч показывает, сколько часов он работает. Переменная e показывает, сколько денег он зарабатывает. Сколько денег заработает Лукас, проработав 6 часов?

Как легко составлять свои собственные математические задачи со словом и рабочие листы с задачами со словом

Вооружившись 120 примерами, чтобы зажечь идеи, создание собственных математических задач со словами может заинтересовать ваших учеников и обеспечить согласованность с уроками. Do:

  • Ссылка на интересы студентов: Обрамляя свои текстовые проблемы интересами студентов, вы, вероятно, привлечете внимание.Например, если большая часть вашего класса любит американский футбол, задача измерения может включать расстояние броска известного квотербека.
  • Задайте тематические вопросы: Написание словесной задачи, отражающей текущие события или проблемы, может заинтересовать учащихся, давая им четкий, осязаемый способ применения своих знаний.
  • Включите имена учащихся: Назовите символы вопроса в честь учащихся — это простой способ сделать предмет более понятным, помогая им справиться с проблемой.
  • Ясно: Повторение ключевых слов определяет вопрос, помогая студентам сосредоточиться на основной проблеме.

Не нужно:

  • Тест на понимание прочитанного: Цветочный выбор слов и длинные предложения могут скрыть ключевые элементы вопроса. Вместо этого используйте краткие фразы и лексику на уровне своего класса.
  • Сосредоточение внимания на схожих интересах: Слишком много вопросов, связанных с интересами, такими как футбол и баскетбол, может оттолкнуть некоторых студентов или оттолкнуть их.
  • Особые опасения: Включение ненужной информации вводит еще один элемент решения проблем, подавляющий многих учеников начальной школы.

Ключ к дифференцированному обучению, словесные задачи, которые студенты могут связать и контекстуализировать, вызовут интерес больше, чем общие и абстрактные.

Заключительные мысли о математических задачах со словами

Скорее всего, вы получите максимальную отдачу от этого ресурса, если будет использовать задачи в качестве шаблонов, слегка изменяя их, применяя приведенные выше советы. Таким образом, они будут более актуальны и интересны для ваших учеников. Тем не менее, наличие 120 задач по математике, соответствующих учебной программе, на кончиках ваших пальцев, должно помочь вам решать задачи по развитию навыков и давать задания, заставляющие задуматься. Результат?
Лучшее понимание того, как ваши ученики обрабатывают контент, и демонстрация понимания, информирующая о вашем текущем подходе к обучению

K-8 Практические задачи по математике — WebMath

Быстро! Мне нужна помощь с:
Выберите пункт справки по математике…Calculus, DerivativesCalculus, IntegrationCalculus, Quotient RuleCoins, CountingCombinations, Find allComplex Numbers, Adding ofComplex Numbers, Calculating withComplex Numbers, MultiplyingComplex Numbers, Powers ofComplex NumberConversion, SubtractingConversion, TemperatureConversion, FindConversion, MassConversion, MassConversion анализ AverageData, поиск стандартного отклонения, анализ данных, гистограммы, десятичные дроби, преобразование в дробь, электричество, стоимость факторинга, IntegerFactors, Greatest CommonFactors, Least CommonFractions, AddingFractions, ComparingFractions, ConvertingFractions, Convert to a decimalFractions, DividingFractions, MultiplyingFractions, SubplicationFractions are, SubplicationFractions , BoxesGeometry, CirclesGeometry, CylindersGeometry, RectanglesGeometry, Right TrianglesGeometry, SpheresGeometry, SquaresGraphing, LinesGraphing, Любая функцияGraphing, CirclesGraphing, EllipsesGraphing, HyperbolasGraphing, InequalitiesGraphing, Polar PlotGraphing, (x, y) pointInequalities, GraphingInequalities, SolvingInterest, CompoundInterest, SimpleLines, The Equation from point and slopeLines, Equation from slope и y-intLines, The Equation from two pointsLodsottery Практика полиномов Математика, Практика основМетрическая система, преобразование чисел, сложение чисел, вычисление с числами, вычисление с переменными Числа, деление чисел, умножение чисел, сравнение числовой линии чисел, числовые строки, размещение значений чисел, произнесение чисел, округление чисел, вычитание параболических чисел, построение чисел в квадрате , Факторинг разности квадратов многочленов, факторинг триномов многочленов, разложение на множители с GCF Полиномы, умножение многочленов, возведение в степень ns, Решить с помощью факторинга Радикалы, Другие корни Радикалы, Отношения квадратного корня, Что они собой представляют, Выведение на пенсию, Экономия на продажной цене, РасчетНаучная нотация, ПреобразованиеНаучной нотации, ДелениеНаучная нотация, Умножение форм, ПрямоугольникиУпрощение, Все, что угодноУпрощение, Образцы, Образцы, Упрощение, Образцы, Упрощение, Пример Правые треугольники, Ветер, рисунок

30 увлекательных вопросов по математике с ответами

Математика может быть интересной, если вы относитесь к ней правильно.Математика — это не что иное, как игра, игра, которая совершенствует ваш интеллект и повышает концентрацию. По сравнению с прежними временами люди стали лучше и дружелюбнее относиться к математике, что делает ее более привлекательной. Золотое правило — знать, что математика — это вдумчивое занятие, а не задача.

Нет ничего лучше сложных математических задач или сложных математических вопросов, просто вы недостаточно хорошо изучили математику, чтобы понять ее легкость и сопоставимость. Сложные математические вопросы и ответы можно превратить в забавные математические задачи, если вы посмотрите на них, как на мозговой штурм.При правильном подходе, а также при наличии друзей и учителей выполнение математики может быть очень увлекательным и увлекательным занятием.

Математика интересна тем, что несколько уравнений и диаграмм могут передавать объемы информации. Относитесь к математике как к языку, переходя к строгому доказательству и используя логическое обоснование для выполнения определенного шага в доказательстве или выводе.

Отношение к математике как к языку полностью исключает представление о сложных математических задачах или сложных математических вопросах из вашего разума. Знакомство детей с забавными математическими вопросами может вызвать у них сильную любовь и признательность к математике в раннем возрасте.Таким образом вы строите успешное будущее ребенка. Веселые математические задачи побудят вашего ребенка решить их, а не играть в бинго или печь.

По-видимому, существует бесчисленное множество способов сделать простые математические сложные вопросы и ответы. Это включает зарождение идеологии, согласно которой математика проще, чем их страхи. Это можно сделать, связав математику с повседневной жизнью. Занятия математикой с помощью игральных костей, карточек, головоломок и таблиц убедят вашего ребенка в том, что он эффективно приближается к математике.

Если вы хотите развлечься и развлечься в образовательной деятельности, также посетите

Веселые математические вопросы с ответами — PDF

Вот загружаемый PDF-файл, который состоит из забавных математических вопросов. Нажмите кнопку «Загрузить», чтобы просмотреть их.

Вот несколько забавных, сложных и сложных математических задач, которые повлияют на ваши мыслительные способности.

  1. Если 1 = 3

2 = 3

3 = 5

4 = 4

5 = 4

Тогда 6 =?

Ответ: равно 3, потому что «шесть» состоит из трех букв

  1. Какое количество парковочных мест занимает автомобиль?

Эта сложная математическая задача стала вирусной несколько лет назад после того, как появилась на вступительном экзамене в Гонконге… для шестилетних детей.Якобы у студентов было всего 20 секунд, чтобы решить задачу!

Ответ:

Хотите верьте, хотите нет, но этот «математический» вопрос вообще не требует математики. (2)

Эта проблема возникла прямо из стандартизированного теста, проведенного в Нью-Йорке в 2014 году.

Ответ:

  1. На выставку зарегистрировано 49 собак. Маленьких собак на 36 больше, чем крупных. Сколько маленьких собак записались на соревнования?

Этот вопрос возникает непосредственно из домашнего задания второклассника по математике.

Ответ:

Чтобы определить, сколько маленьких собак соревнуются, вы должны вычесть 36 из 49, а затем разделить ответ 13 на 2, чтобы получить 6.5 собак, или количество соревнующихся крупных собак. Но вы еще не закончили! Затем вам нужно добавить 6,5 к 36, чтобы получить количество соревнующихся маленьких собак, которое составляет 42,5. Конечно, на самом деле половина собаки не может участвовать в выставках собак, но ради решения этой математической задачи предположим, что это так.

  1. Складываем 8,563 и 4,8292.

Ответ:

Сложить два десятичных знака проще, чем кажется.Пусть вас не сбивает с толку тот факт, что в 8,563 меньше чисел, чем в 4,8292. Все, что вам нужно сделать, это добавить 0 в конец 8.563, а затем добавить, как обычно.

  1. Я нечетное число. Убери одну букву и я стану ровным. Какой я номер?

Ответ: Семь (уберите «s», и оно станет «четным»).

  1. Используя только сложение, как сложить восемь восьмерок и получить число 1000?

Ответ:

888 + 88 + 8 + 8 + 8 = 1000

  1. Салли 54 года, а ее матери 80 лет, сколько лет назад мать Салли была больше ее возраста?

Ответ:

41 год назад, когда Салли было 13, а ее матери — 39.

  1. Какие 3 числа имеют одинаковый ответ независимо от того, сложены они или умножены вместе?

Ответ:

  1. Есть корзина с 5 яблоками. Как разделить яблоки между 5 детьми, чтобы у каждого ребенка было по 1 яблоку, а в корзине осталось 1 яблоко?

Ответ:

Четверо детей получают по 1 яблоку, а пятый ребенок получает корзину с оставшимся яблоком.

  1. Это трехзначное число. Вторая цифра в четыре раза больше третьей цифры, а первая цифра на три меньше второй. Какой номер?

Ответ:

  1. Заполните вопросительный знак

Ответ:

  1. Две девочки родились от одной матери, в одно и то же время, в один день, в один месяц и один год, но почему-то они не близнецы.Почему нет?

Ответ:

Потому что была третья девочка, которая делает их тройняшками!

  1. Корабль, стоящий на якоре в порту, имеет лестницу, свисающую с борта. Длина лестницы — 200 см, расстояние между каждой ступенькой 20 см, нижняя ступенька касается воды. Скорость прилива составляет 10 см в час. Когда вода достигнет пятой ступеньки?

Ответ:

Прилив поднимает и воду, и лодку, поэтому вода никогда не достигает пятой ступени.

  1. Позавчера мне было 25. В следующем году мне будет 28. Это правда только один день в году. Какой день у меня День Рождения?

Ответ:

  1. У вас есть 3-литровая бутылка и 5-литровая бутылка. Как можно отмерить 4 литра воды из бутылок на 3 и 5 литров?

Ответ:

Решение 1:

  • Сначала наполните 3-литровую бутылку и налейте 3 литра в 5-литровую бутылку.

  • Снова наполните 3-литровую бутылку. Теперь налейте 2 литра в 5-литровую бутылку, пока она не станет полной.

  • Теперь пустая 5-литровая бутылка.

  • Налейте оставшийся 1 литр из 3-литровой бутылки в 5-литровую.

  • Теперь снова наполните 3-литровую бутыль и налейте 3 литра в 5-литровую бутыль.

  • Теперь у вас 4 литра в 5-литровой бутылке.Это оно.

Решение 2:

  • Сначала наполните 5-литровую бутылку и налейте 3 литра в 3-литровую бутылку.

  • Пустая 3-литровая бутылка.

  • Вылейте оставшиеся 2 литра 5-литрового баллона в 3-литровый.

  • Снова наполните 5-литровую бутылку и налейте 1 литр в 3-литровую бутылку, пока она не станет полной.

  • Теперь у вас 4 литра в 5-литровой бутылке.Это оно.

  1. 3 друга пошли в магазин и купили 3 игрушки. Каждый человек заплатил 10 рупий, что составляет стоимость одной игрушки. Итак, они заплатили 30 рупий, то есть всю сумму. Владелец магазина предоставил скидку в размере 5 рупий на покупку 3 игрушек за 30 рупий. Затем, среди 5 рупий, каждый человек взял 1 рупий, а оставшиеся 2 рупии передал нищему рядом с магазином. Теперь эффективная сумма, выплачиваемая каждым человеком, составляет 9 рупий, а сумма, выплачиваемая нищему, — 2 рупия. Таким образом, общая эффективная выплаченная сумма составляет 9 * 3 = 27, а сумма, отданная нищему, составляет рупий.2, то есть общая сумма составляет 29 рупий. Куда пропал другой рупий по сравнению с первоначальными 30 рупиями?

Ответ:

Логика в том, что платежи должны быть равны поступлениям. Мы не можем сложить сумму, уплаченную отдельными лицами, и сумму, отданную нищему, и сравнить ее с 30 рупиями. Общая выплаченная сумма составляет 27 фунтов стерлингов. Итак, с 27 владелец магазина получил 25 рупий, а нищий получил 2. Таким образом, выплаты приравниваются к поступлениям.

  1. Как получить число 100, используя четыре семерки (семерки) и одну (1)?

Ответ 1: 177 — 77 = 100;

Ответ 2: (7 + 7) * (7 + (1/7)) = 100

  1. Переместите любые четыре спички, чтобы получить только 3 равносторонних треугольника (спички не удаляйте)

Ответ:

  1. Найдите площадь красного треугольника.

Ответ:

Чтобы решить этот забавный математический вопрос, вам нужно понять, как работает площадь параллелограмма. Если вы уже знаете, как связаны площадь параллелограмма и площадь треугольника, тогда сложение 79 и 10 и последующее вычитание 72 и 8 для получения 9 должно иметь смысл

  1. Сколько футов в миле?

Ответ:

  1. Решить — 15+ (-5x) = 0

Ответ:

  1. Что такое 1.92 ÷ 3

Ответ:

  1. Мужчина поднимается на гору наклонную. Ему нужно преодолеть 100 км, чтобы достичь вершины горы. Каждый день днем ​​он поднимается на 2 км вперед. Измученный, он отдыхает там ночью. Ночью, когда он спит, он скатывается на 1 км назад, потому что гора наклонная. Тогда сколько дней ему понадобится, чтобы достичь вершины горы?

Ответ:

  1. Если 72 x 96 = 6927, 58 x 87 = 7885, то 79 x 86 =?

Ответ:

  1. Посмотрите на этот ряд: 36, 34, 30, 28, 24,… Какое число должно быть следующим?

Ответ:

  1. Посмотрите на этот ряд: 22, 21, 23, 22, 24, 23,… Какое число должно быть следующим?

Ответ:

  1. Если 13 x 12 = 651 и 41 x 23 = 448, тогда 24 x 22 =?

Ответ:

  1. Посмотрите на этот ряд: 53, 53, 40, 40, 27, 27,… Какое число должно быть следующим?

Ответ:

Читайте также:


Заключение

Конечная цель обучения математике — понимание учащимися представленного материала, применение навыков и вспоминание концепций в будущем.Мало пользы от того, что студенты вспомнят формулу или процедуру для подготовки к завтрашнему экзамену, а к следующей неделе забывают основную концепцию.

Учителя должны сосредоточиться на том, чтобы ученики понимали материал, а не просто запоминали процедуры. Узнав ответы на забавный математический вопрос, вы начинаете спрашивать себя, как вы могли пропустить что-то такое простое. По правде говоря, большинство хитрых вопросов созданы для того, чтобы обмануть ваш разум, поэтому ответы на забавные математические вопросы логичны и просты.


О компании Cuemath

Cuemath, удобная для учащихся платформа математики и кодирования, проводит регулярные онлайн-классы для преподавателей и развития навыков, а их приложение Mental Math для iOS и Android представляет собой универсальное решение для детей, развивающее несколько навыков. Ознакомьтесь со структурой Cuemath Fee и подпишитесь на бесплатную пробную версию.

Бесплатные задания по математике | по Math-Drills

Math-Drills.com включает более 58 тысяч бесплатных рабочих листов по математике , которые могут быть использованы, чтобы помочь учащимся изучать математику.Наши рабочие листы по математике доступны по широкому кругу тем, включая определение чисел, арифметику, предварительную алгебру, геометрию, измерения, денежные концепции и многое другое. Есть две интерактивные математические функции: математические карточки и математическая игра с точками.

Math-Drills.com был запущен в 2005 году и содержит около 400 рабочих листов по математике. С тех пор были добавлены еще десятки тысяч математических листов. Веб-сайт и контент продолжают улучшаться на основе отзывов и предложений наших пользователей, а также наших собственных знаний об эффективных математических методах.

Большинство пользователей Math-Drills — классные учителя или родители. Классные учителя используют наши рабочие листы по математике для оценки усвоения учащимися основных математических фактов, чтобы дать учащимся дополнительную практику по математике, обучить новым математическим стратегиям и сэкономить драгоценное время на планирование. Родители используют наши рабочие листы по математике, чтобы дать своим детям дополнительную практику по математике во время школьных каникул и улучшить их математическое образование. Домашние школы используют наши рабочие листы по математике в своих программах, чтобы развивать и укреплять математические навыки у своих детей.

Самые популярные бесплатные задания по математике на этой неделе

Поддержка обучения учащихся с помощью бесплатных заданий по математике с помощью Math-Drills

Math-Drills считает, что образование должно быть доступным для всех детей, независимо от их социально-экономического положения или любых других факторов. С момента его появления в 2005 году все математические рабочие листы на Math-Drills были бесплатными для студентов, изучающих математику. Веб-сайт Math-Drills хорошо работает на любом устройстве, и рабочие листы можно распечатать или использовать на экране.

Рабочие листы

Math-Drills также используются в специальном образовании, обучении взрослых, репетиторстве, колледжах, средних школах, тюрьмах и во многих других ситуациях. Учителям специального образования особенно нравится то, что мы разбиваем математические навыки и предоставляем варианты с крупным шрифтом. Взрослые учащиеся ценят простой и лаконичный формат, который предлагают наши рабочие листы. Репетиторы используют наши рабочие листы по математике, чтобы сократить свои расходы и сосредоточиться на обучении студентов. Преподавателям средней школы, колледжа и университета иногда требуются дополнительные ресурсы для учащихся, чтобы они могли продолжить изучение более сложных тем.

Учащиеся, которые практикуют свои математические навыки с нашими заданиями по математике во время школьных каникул, сохранят свои математические навыки на высоком уровне к предстоящим учебным семестрам. Поскольку мы предоставляем ключи ответов, учащиеся могут самостоятельно оценивать и использовать немедленную обратную связь, предоставляемую ключом ответов, для анализа и исправления ошибок в своей работе. Наши интерактивные (заполняемые) математические листы позволяют им заполнять свои ответы на экране и сохранять или распечатывать результаты.

Присоединяйтесь к информационному бюллетеню Math-Drills

Подпишитесь на нашу рассылку, чтобы узнавать о новых рабочих листах по математике и другой информации, связанной с сайтом.

Еще 4 трудных задачи по математике, которые заставят вас почувствовать, что вам нужно вернуться в начальную школу

Помните тот раз, когда мы полностью поставили вас в тупик пятью, казалось бы, простыми математическими задачами, которые на самом деле запутали ваш мозг в узел? Что ж, мы снова за это.

Вот еще четыре супер простых задачи, которые на самом деле сбивают вас с толку!

1. Тест на добавление скорости

Давайте начнем легко. Сложите следующие числа сверху вниз как можно быстрее в уме.

Получили 5000? Что ж, это было бы … неправильно.

Ответ: 4100

Объяснение: Это простой случай, когда ваш мозг опережает самого себя. У вас были все шансы на успех, пока вы не добрались до последнего добавления.

1000 + 20 = 1020 (справа)

1020 + 30 = 1050 (Тотал.)

1050 + 1000 = 2050 (Ага)

2050 + 1030 = 3080 (Ммммм.)

3080 + 1000 = 4080 (Да, почти готово!)

4080 + 20 = 4100

Эээээ, что ?! Вы определенно не понимали этого раньше.

Сейчас кажется совершенно очевидным, что все это делается медленно у вас на глазах, но что заставило вас промахнуться с этим последним дополнением в первый раз, так это то, что когда вы быстро складывали все в своей голове, вам никогда не приходилось носить их с собой. один до самого конца, и когда вам, наконец, нужно нести один, вы случайно добавили его к тысячам, а не к сотням, потому что вы двигались так быстро.Или, может быть, вы не заметили 30 из 1030 предпоследней строки.

Или, может быть, вы просто гений и были правы с первого раза, и тогда вы молодцы!

2. Кто ремонтировал ваш неисправный водонагреватель?

Предположим, у вас сломался водонагреватель, и вы не можете принять горячий душ. Вы идете к человеку и просите его проверить ваш водонагреватель. Этот человек приходит к вам домой и использует кучу запчастей, а затем ремонтирует их, так что вы платите ему или ей за ремонт. Этот человек более вероятен:

Бухгалтер?

ИЛИ

Бухгалтер и сантехник.

Getty Images

Вы ответили сантехнику? Это понятно, но вы ошибаетесь.

Ответ: Человек, скорее всего, бухгалтер.

Объяснение: Когда вы прочитали это слово «проблема», вы интуитивно пришли к выводу, что этот человек, скорее всего, был водопроводчиком, потому что, ну, сантехники ремонтируют водонагреватели. НО вопрос спрашивает, что более вероятно, что означает, что это вопрос вероятности.

Строго говоря, это скорее бухгалтер, чем сантехник. Здесь важно помнить, что вопрос заключается в том, что человек, который ремонтирует обогреватель, скорее всего, бухгалтер или бухгалтер и сантехник (также известный как сантехник-бухгалтер).

Итак, вероятности таковы, что

[A] сантехник-бухгалтер отремонтировал ваш обогреватель (вероятно, очень маленький, верно? Немногие люди одновременно являются водопроводчиками и бухгалтерами),

[B] бухгалтер отремонтировал ваш обогреватель (вероятно, бухгалтеров больше, чем бухгалтеров-сантехников)

Кроме того, в этой ситуации любой сантехник определенно является бухгалтером, поэтому вы фактически складываете эти вероятности вместе.

А ≤ А + В

или

Бухгалтеры-сантехники ≤ Бухгалтер-сантехники + бухгалтеры

Значит, скорее всего, бухгалтер!

3. Каков ответ на следующее уравнение?

Вы сначала умножили 1 x 0, а затем сложили остальные и получили 12? Неправильный!

Ответ: Ответ — 2. Да, 2!

Объяснение: Поскольку в конце каждой строки нет символов операторов (+, -, x, /), нет математических оснований полагать, что каждая строка является частью одного и того же уравнения.А поскольку уравнение — это утверждение, что значения двух математических выражений равны, поскольку на концах первых двух строк нет знаков равенства, это вовсе не уравнения. Это просто выражения. Это означает, что единственное уравнение на картинке выше — это последняя строка и:

1 + 1 х 0 + 1 = 2

Некоторые утверждают, что вы должны соединить строки вместе, сделав две строки в конце каждой строки 11, и в этом случае ответ будет 30, потому что:

1 + 1 + 1 + 1 + 11 + 1 + 1 + 1 + 11 + 1 х 0 + 1 = 30

Но это не совсем математически корректно, потому что математика не похожа на английский.Вы не просто «продолжаете читать» до следующей строки (это вызовет много путаницы и двусмысленности в математических задачах). Если у вас есть длинное уравнение, которое необходимо разбить на несколько строк, разрыв строки должен располагаться непосредственно перед или после символа оператора. Итак, если 30 должно было быть ответом, проблема должна была быть написана:

1 + 1 + 1 + 1 +

11 + 1 + 1 + 1

+ 11 + 1 х 0 +1 = 30

4. Значение 0,999 … равно?

Вот простой вопрос:

Ну что ставить знают все… в конце трех девяток подряд означает, что 9 продолжается бесконечно, поэтому вы ответили ложно. 0,999 … никогда не может равняться 1, верно?

Ответ: Нет. Неправильный. Фактически, он равен единице. Вот доказательство, подтверждающее это:

Объяснение: Причина, по которой это так трудно понять, заключается в том, что концепция бесконечности изначально сложна для понимания. Большинство людей просто воображают, что где-то в конце концов есть последние 9.Но дело в том, что девятки бесконечны.

Также важно помнить, что то, что два числа выглядят по-разному, не означает, что они не одно и то же значение. 0,5 — это определенно то же, что и 1/2. А 2 + 2 то же самое, что 4. А 0,999 … абсолютно равно 1. Это просто два разных способа выражения одного и того же значения.

Вот еще одно, еще более простое доказательство, которое может помочь вам понять. Мы все согласны с тем, что 1/3 = 0,333 … повторение, верно? Что ж, посмотри:

Смотрите! На самом деле это довольно просто! Если вам все еще трудно понять это, позвольте эксперту по объяснению на YouTube, ВиХарт, объяснить вам это во всех деталях (и с забавными каракулями).

Этот контент импортирован с YouTube. Вы можете найти тот же контент в другом формате или найти дополнительную информацию на их веб-сайте.

Ноэль Дево
Редактор развлечений
Когда я не запираюсь в своей комнате из-за совершенно непродуктивного запоя Netflix или из-за преследования Тимоти Шаломе в Tumblr, я ищу потрясающие новости о знаменитостях, которые понравятся читателям Seventeen!

Этот контент создается и поддерживается третьей стороной и импортируется на эту страницу, чтобы помочь пользователям указать свои адреса электронной почты.

Добавить комментарий

Ваш адрес email не будет опубликован. Обязательные поля помечены *